QUIZ 3 PHARM

Ace your homework & exams now with Quizwiz!

The nurse is discussing upper and lower UTIs. Which of these conditions can be treated at home? (Select all that apply.) a. Acute cystitis b. Acute urethritis c. Recurrent UTI d. Severe pyelonephritis e. Acute bacterial prostatitis

A, B, C Acute cystitis, urethritis, and recurrent UTIs can be treated on an outpatient basis. Severe pyelonephritis and acute bacterial prostatitis require hospitalization and IV antibiotics.

Nitrofurantoin [Macrodantin] is prescribed for an adolescent female patient with acute cystitis. What should the nurse include in the teaching for this patient? (Select all that apply.) a. "Make sure you tell your prescriber if you might be pregnant." b. "If you experience any tingling or numbness, stop taking the drug and call the clinic immediately." c. "Headaches and drowsiness can occur and are mild side effects." d. "You should not take sulfonamides while taking this medication." e. "Your urine may have a brown tinge while you are taking this drug."

A, B, C, E Nitrofurantoin is linked to serious birth defects and is not recommended during pregnancy. Tingling and numbness indicate neuropathy, which is a serious and often irreversible side effect. Central nervous system (CNS) side effects usually are mild and reversible. Sulfonamides cannot be taken with methenamine, because they can cause crystalluria. Patients' urine may have a brown tinge while they are taking this drug.

The nurse is teaching a pharmacology refresher course to a group of nurses. A student asks what host factors affect the choice of agents in antimicrobial therapy. The nurse will tell the students that such host factors include what? (Select all that apply.) a. Age b. Gender c. Immune system status d. Infection site e. Weight

A, C, D

A patient who is taking nelfinavir [Viracept] calls the nurse to report moderate to severe diarrhea. What will the nurse expect the provider to recommend? a.An over-the-counter antidiarrheal drug b.Immediate discontinuation of the nelfinavir c.Reducing the dose of nelfinavir by half d.Taking the nelfinavir with food to avoid side effects

ANS: A A dose-limiting effect of nelfinavir is moderate to severe diarrhea, which can be managed with OTC antidiarrheal medications. Unless the symptoms become severe, withdrawing the nelfinavir is not indicated. Reducing the dose by half or taking it with food is not indicated.

A patient with a urinary tract infection is given a prescription for TMP/SMZ. When reviewing the drug with the patient, the nurse learns that the patient has type 1 diabetes mellitus and consumes alcohol heavily. What will the nurse do? a. Contact the provider to request a different antibiotic for this patient. b. Obtain frequent blood glucose determinations while giving TMP/SMZ. c. Suggest that the patient take a potassium supplement while taking TMP/SMZ. d. Tell the patient to avoid excessive fluid intake while taking TMP/SMZ.

ANS: A Alcoholics are likely to be folate deficient and have an increased risk of megaloblastic anemia when taking TMP/SMZ, so withholding this drug in this population is recommended. TMP/SMZ shares hypersensitivity reactions with oral sulfonylurea-type hypoglycemics that are used with type 2 diabetes mellitus, so it is not necessary to assess the blood glucose level more often. TMP/SMZ can cause hyperkalemia, so potassium supplements are contraindicated. Patients taking TMP/SMZ should consume more fluids to maintain renal blood flow and prevent renal damage.

A patient with HIV and mucocutaneous HSV is being treated with foscarnet after failing treatment with acyclovir. After 2 weeks, the patient's dose is increased to 90 mg/kg over 2 hours from 40 mg/kg over 1 hour. The patient reports numbness in the extremities and perioral tingling. What will the nurse do? a.Notify the provider and request an order for a serum calcium level. b.Notify the provider of potential foscarnet overdose. c.Request an order for a creatinine clearance level. d.Request an order of IV saline to be given before the next dose.

ANS: A Foscarnet frequently causes hypocalcemia and other electrolyte and mineral imbalances. Paresthesias, numbness in the extremities, and perioral tingling can indicate hypocalcemia, so a calcium level should be drawn. These are not signs of foscarnet overdose. Nephrotoxicity may occur, but these are not signs of renal complications, so a creatinine clearance is not indicated. If nephrotoxicity occurs, prehydration with IV saline is indicated to reduce the risk of renal injury.

A patient has lamivudine-resistant hepatitis B and has been taking entecavir [Baraclude] for 2 years. The patient asks the nurse why the provider has recommended taking the drug for another year. What will the nurse tell the patient? a."Entecavir can reverse fibrosis and cirrhosis of the liver when taken long term." b."It is necessary to continue taking entecavir to avoid withdrawal symptoms." c."The drug will be given until the infection is completely eradicated." d."You will need to continue taking entecavir to prevent lactic acidosis and hepatotoxicity."

ANS: A Recent evidence indicates that, with long-term use (3 years), entecavir can reverse fibrosis and cirrhosis. The drug is not continued to avoid withdrawal symptoms. Patients who stop taking entecavir may experience acute exacerbations of hepatitis B; the disease is not eradicated. Entecavir can cause lactic acidosis and hepatotoxicity; it does not prevent these adverse effects.

A patient will be discharged from the hospital with a prescription for TMP/SMZ [Bactrim]. When providing teaching for this patient, the nurse will tell the patient that it will be important to: a. drink 8 to 10 glasses of water each day. b. eat foods that are high in potassium. c. take the medication with food. d. take folic acid supplements.

ANS: A TMP/SMZ can injure the kidneys, because it causes deposition of sulfonamide crystals in the kidneys. Patients should be advised to drink 8 to 10 glasses of water a day to maintain a urine flow of 1200 mL in adults. Trimethoprim can cause hyperkalemia, so consuming extra potassium is unnecessary. The medication should be taken on an empty stomach. It is not necessary to consume extra folic acid, because mammalian cells use dietary folate and do not have to synthesize it; it is the process of folic acid synthesis that is altered by sulfonamides.

The nurse is caring for a patient who is HIV positive and has a previous history of drug and alcohol abuse. The patient is being treated with combination therapies, including didanosine [Videx]. Which laboratory findings would most concern the nurse? a.Increased serum amylase and triglycerides and decreased serum calcium b.Decreased serum amylase and serum triglycerides and increased serum calcium c.Decreased hemoglobin and hematocrit d.Increased serum amylase, decreased triglycerides, and increased platelets

ANS: A The nurse should be concerned about increased serum amylase triglycerides and a decreased serum calcium, which are symptoms of pancreatitis, the major adverse effect of didanosine. The other laboratory test results and assessment findings are not consistent with pancreatitis and are not a concern for the nurse.

The nurse is caring for a patient on a medical-surgical unit who has been experiencing fever of unknown origin. The prescriber has ordered a broad-spectrum antibiotic. Which intervention is the priority? a. Administer the antibiotic immediately. b. Administer antipyretics as soon as possible. c. Obtain all cultures before the antibiotic is administered. d. Delay administration of the antibiotic until the culture results are available.

C It is essential that samples of exudates and body fluids (in this case, blood cultures) be obtained for culture before initiation of treatment. Administration of the antibiotic is important but not more important than obtaining specimens for culture. Anti-pyretics may be indicated, but the priority is obtaining specimens for culture. Treatment may be initiated before the test results are available

A patient is being treated with amphotericin B [Abelcet] for a systemic fungal infection. After several weeks of therapy, the provider orders flucytosine [Ancobon] in addition to the amphotericin. The nurse understands that the rationale for this combination is that it: a. broadens the antifungal spectrum. b. improves the effectiveness of the amphotericin B. c. lowers the dose of amphotericin B and reduces toxicity. d. treats fungal central nervous system (CNS) infection.

C Amphotericin potentiates the antifungal actions of flucytosine, and combining flucytosine with low-dose amphotericin can produce antifungal effects equivalent to those of high-dose amphotericin. By allowing a reduced dose of amphotericin, the combination also reduces the risk of amphotericin-induced toxicity. Flucytosine does not affect the effectiveness of amphotericin. The combination does not broaden the antifungal spectrum or alter the ability to treat CNS infections.

A nurse is discussing intravenous amphotericin B treatment with a nursing student who is about to care for a patient with a systemic fungal infection. Which statement by the student indicates a need for further teaching? a. "A test dose of amphotericin B may be given to assess the patient's reaction." b. "If I see any precipitate in the IV solution, I should stop the infusion immediately." c. "Infusions of amphotericin B should be administered over 1 to 2 hours." d. "The IV site should be rotated frequently to reduce the risk of phlebitis."

C Infusions of amphotericin B should be given over 2 to 4 hours to minimize phlebitis and cardiovascular reactions; therefore, 1 to 2 hours is incorrect. Test doses are given to evaluate the patient's response. Any precipitate in the IV solution warrants immediate discontinuation of the IV. IV sites should be rotated frequently

Which patients should be given antibiotics prophylactically? a. children who attend day care with other children who have strep throat b. children undergoing chemotherapy who have mild neutropenia c. patients with certain congenital heart defects at risk for bacterial endocarditis d. patients with compound fractures undergoing surgical repair e. postoperative patient who have undergone emergency cesarean sections

C, D

Which are benefits of using a combination of two or more antibiotics? (Select all that apply.) a. Reduced cost b. Reduced risk of superinfection c. Reduced toxicity d. Reduced resistance e. Reduced risk in severe infection

C, D, E

A nurse is caring for a patient who has a superficial fungal infection and notes that the provider has ordered 200 mg of ketoconazole [Nizoral] 200 mg PO once daily. Which action by the nurse is correct? a. Administer the drug as ordered. b. Contact the provider to discuss twice daily dosing. c. Discuss a 400-mg daily drug regimen with the provider. d. Request an order for a different antifungal medication.

D Because of the risk of serious and potentially fatal hepatic necrosis with oral ketoconazole, it is not recommended for use in treating superficial fungal infections. Another antifungal medication should be ordered

A healthcare worker who is asymptomatic has a screening TST result of 10 mm of induration during a pre-employment physical. What will the nurse reading this test tell the patient? a. "This is a negative test, so you are cleared for employment." b. "You have latent TB and will need to take isoniazid for 6 to 9 months." c. "You need to have a chest radiograph and a sputum culture." d. "You will begin taking a four-drug regimen to treat tuberculosis."

c. "You need to have a chest radiograph and a sputum culture."

A patient has been taking isoniazid [Nydrazid] for 4 months for latent tuberculosis. The patient reports bilateral tingling and numbness of the hands and feet, as well as feeling clumsy. The nurse expects the provider to: a. discontinue the isoniazid. b. lower the isoniazid dose and add rifampin. c. order pyridoxine 100 mg per day. d. recheck the tuberculin skin test to see whether it worsens.

c. order pyridoxine 100 mg per day.

A patient who is being treated for HIV infection has a 5-mm area of induration after a routine TST. The patient's chest radiograph is normal, and there are no other physical findings. The nurse will expect this patient to begin treatment with which drugs? a. Isoniazid and rifabutin b. Isoniazid and rifampin c. Isoniazid and rifapentine d. Isoniazid and pyrazinamide

a. Isoniazid and rifabutin

A patient with HIV who takes protease inhibitors develops tuberculosis and will begin treatment. Which drug regimen will be used for this patient? a. Isoniazid, pyrazinamide, ethambutol + rifabutin b. Isoniazid, pyrazinamide, ethambutol c. Isoniazid, rifampin, pyrazinamide, ethambutol d. Isoniazid + rifabutin

a. Isoniazid, pyrazinamide, ethambutol + rifabutin

A patient with high-risk factors for tuberculosis will begin therapy for latent TB with isoniazid and rifampin. The nurse learns that this patient takes oral contraceptives. The nurse will counsel this patient to discuss ____ with her provider. a. another birth control method b. reducing the rifampin dose c. reducing the isoniazid dose d. increasing the oral contraceptive dose

a. another birth control method

A patient newly diagnosed with tuberculosis asks the nurse why oral medications must be given in the clinic. The nurse will tell the patient that medications are given in the clinic so that: a. clinic staff can observe adherence to drug regimens. b. nurses can monitor for drug toxicities. c. providers can adjust doses as needed. d. the staff can ensure that the U.S. Food and Drug Administration (FDA) regulations are met.

a. clinic staff can observe adherence to drug regimens.

A hospitalized patient is being treated for tuberculosis with a drug regimen that includes pyrazinamide. The patient complains of pain in the knee and shoulder joints. The nurse will contact the provider to request an order for: a. ibuprofen. b. renal function tests. c. discontinuation of the pyrazinamide. d. measurement of uric acid levels.

a. ibuprofen.

A patient is receiving tobramycin 3 times daily. The provider has ordered a trough level with the 8:00 AM dose. The nurse will ensure that the level is drawn at what time? a. 4:00 AM b. 7:00 AM c. 7:45 AM d. 8:45 AM

ANS: C When a patient is receiving divided doses of an aminoglycoside, the trough level should be drawn just before the next dose; therefore, 7:45 AM would be the appropriate time. It would not be appropriate to draw a trough at the other times listed.

15. A patient develops CDAD. Which antibiotic is recommended for treating this infection? a. Chloramphenicol b. Clindamycin [Cleocin] c. Linezolid [Zyvox] d. Vancomycin

ANS: D Vancomycin and metronidazole are the drugs of choice for treating CDAD.

A patient is about to begin therapy with ethambutol. The nurse knows that, before initiating treatment with this drug, it is important to obtain which test(s)? a. Color vision and visual acuity b. Complete blood cell (CBC) count c. Hearing testing and a tympanogram d. Hepatic function tests

a. Color vision and visual acuity

Which superficial mycosis is generally treated with oral antifungal agents? a. Tinea capitis b. Tinea corporis c. Tinea cruris d. Tinea pedis

A Tinea capitis must be treated with oral agents for 6 to 8 weeks. Tinea corporis, tinea cruris, and tinea pedis may be treated topically

A patient is being treated for a systemic fungal infection with amphotericin B [Abelcet] and will be discharged home from the hospital to complete every other day infusions of the medication for 6 to 8 weeks. The nurse provides discharge teaching before dismissal. Which statement by the patient indicates a need for further teaching? a. "I may experience headaches and pain in my lower back, legs, and abdomen." b. "I may need to take potassium supplements while taking this drug." c. "I should take acetaminophen and diphenhydramine before each infusion." d. "I will need to have blood drawn for serum creatinine and BUN levels every 3 to 4 days."

A Headaches, lower back pain, leg pain, and abdominal pain occur with intrathecal administration of amphotericin B. Patients taking amphotericin may experience hypokalemia and may need potassium supplements. Acetaminophen and diphenhydramine should be taken to minimize infusion reaction effects. Renal function should be monitored every 3 to 4 days during treatment

The nurse is preparing to administer amphotericin B intravenously. The nurse will expect to pretreat the patient with which medications? a. Acetaminophen [Tylenol], diphenhydramine [Benadryl], and meperidine [Demerol] b. Aspirin, diphenhydramine [Benadryl], and meperidine [Demerol] c. Ibuprofen [Motrin], diphenhydramine [Benadryl], and meperidine [Demerol] d. Morphine sulphate [Morphine] and acetaminophen ]Tylenol]

A Optimum pretreatment before the administration of amphotericin B comprises acetaminophen, diphenhydramine, and meperidine (for rigors). Motrin is not suggested as pretreatment. Aspirin is an option, but it may increase kidney damage. Morphine is not indicated in the pretreatment regimen

The nurse has been caring for a patient who has been taking antibiotics for 3 weeks. Upon assessing the patient, the nurse notices the individual has developed oral thrush. Which of the following describes the etiology of the thrush? a. Suprainfection b. Antibiotic resistance c. Nosocomial infection d. Community-acquired infection

A Oral thrush is a manifestation of a superinfection. The development of thrush is not a symptom of antibiotic resistance. Oral thrush typically is not a community-acquired infection. The development of thrush is not a nosocomial infection.

A patient has a systemic candidal infection, and the provider has ordered oral fluconazole [Diflucan] 400 mg on day 1 and 200 mg once daily thereafter. What will the nurse do? a. Administer the drug as ordered. b. Contact the provider to discuss cutting the dosing in half. c. Contact the provider to discuss giving 400 mg on all days. d. Contact the provider to discuss giving the drug intravenously.

A The dosing for fluconazole for systemic candidal infections is 400 mg on the first day and 200 mg/day each day thereafter. Dosing for oral or esophageal candidal infections is half that for systemic infections. It is not correct to give 400 mg on all days of therapy. Fluconazole is as effective when given orally as it is when given intravenously.D

A pregnant patient with fever, flank pain, and chills has a history of two previous bladder infections before getting pregnant. She is allergic to several antibiotics. She reports having taken methenamine successfully in the past. What will the nurse tell her? a. "This agent is not effective against infections of the upper urinary tract." b. "This antiseptic agent is safe for use during pregnancy and has no drug resistance." c. "This drug is linked to many serious birth defects and is not recommended during pregnancy." d. "You will need to take this medication with meals to avoid gastric upset."

A Methenamine is safe for use during pregnancy and would be an excellent choice for this patient if she had a lower urinary tract infection. However, it is not an effective agent for upper urinary tract infection, because it is a prodrug that must break down into ammonia and formaldehyde to be effective. There is not enough time for formaldehyde to form in the kidneys, so it is not effective in the upper tract. Nitrofurantoin is linked to serious birth defects and also must be given with food to prevent gastrointestinal problems.

Before giving methenamine [Hiprex] to a patient, it is important for the nurse to review the patient's history for evidence of which problem? a. Elevated blood urea nitrogen and creatinine b. History of reactions to antibiotic agents c. Possibility of pregnancy d. Previous resistance to antiseptic agents

A Methenamine should not be given to patients with renal impairment, because crystalluria can occur. There is no cross-reactivity between methenamine and antibiotic agents. Methenamine is safe for use during pregnancy. There is no organism drug resistance to methenamine.

A patient has a positive urine culture 1 week after completion of a 3-day course of antibiotics. The nurse anticipates that the prescriber will: a. begin a 2-week course of antibiotics. b. evaluate for a structural abnormality of the urinary tract. c. initiate long-term prophylaxis with low-dose antibiotics. d. treat the patient with intravenous antibiotics.

A Patients who develop a subsequent urinary tract infection after treatment are treated in a stepwise fashion, beginning with a longer course of antibiotics. The next steps would be to begin a 4- to 6-week course of therapy, followed by a 6-month course of therapy if that is unsuccessful. If urinary tract infections are thought to be caused by other complicating factors, an evaluation for structural abnormalities may be warranted. Unless the infections are severe or are complicated, intravenous antibiotics are not indicated.

A patient with a history of renal calculi has fever, flank pain, and bacteriuria. The nurse caring for this patient understands that it is important for the provider to: a. begin antibiotic therapy after urine culture and sensitivity results are available. b. give prophylactic antibiotics for 6 weeks after the acute infection has cleared. c. initiate immediate treatment with broad-spectrum antibiotics. d. refer the patient for intravenous antibiotics and hospitalization.

A Patients with renal calculi are more likely to have complicated urinary tract infections that have less predictable microbiologic etiologies. Because the symptoms are mild, it is important first to obtain a culture and sensitivity to assist with antibiotic selection. If symptoms worsen, a broad-spectrum antibiotic may be started until sensitivity information is available. Intravenous antibiotics are indicated for severe pyelonephritis. Long-term prophylaxis is not indicated unless this patient develops frequent reinfection.

A 20-year-old female patient has suprapubic discomfort, pyuria, dysuria, and bacteriuria greater than 100,000/mL of urine. Which are the most likely diagnosis and treatment? a. Uncomplicated lower urinary tract infection treatable with short-course therapy b. Complicated lower urinary tract infection treatable with single-dose therapy c. Uncomplicated upper urinary tract infection requiring 14 days of oral antibiotics d. Complicated upper urinary tract infection requiring parenteral antibiotics

A These are symptoms of uncomplicated cystitis, which is a lower urinary tract infection that can be treated with a short course of antibiotics. Short-course therapy is more effective than single-dose therapy and is preferred. A complicated lower urinary tract infection would be associated with some predisposing factor, such as renal calculi, an obstruction to the flow of urine, or an indwelling catheter. Upper urinary tract infections often include severe flank pain, fever, and chills.

The nurse is caring for a patient receiving intravenous acyclovir [Zovirax]. To prevent nephrotoxicity associated with intravenous acyclovir, the nurse will: a.hydrate the patient during the infusion and for 2 hours after the infusion. b.increase the patient's intake of foods rich in vitamin C. c.monitor urinary output every 30 minutes. d.provide a low-protein diet for 1 day before and 2 days after the acyclovir infusion.

ANS: A The nurse should ensure that the patient is hydrated during the acyclovir infusion and for 2 hours after the infusion to prevent nephrotoxicity. Increasing vitamin C would not help prevent nephrotoxicity. Monitoring urine output is important but would not help prevent nephrotoxicity. A low-protein diet is not indicated after an acyclovir infusion.

A patient comes to the clinic and receives valacyclovir [Valtrex] for a herpes-zoster virus. The nurse instructs the patient to take the medication: a.without regard to meals. b.without any dairy products. c.each morning. d.on an empty stomach.

ANS: A The patient may take the medication without regard to meals. The patient does not need to avoid dairy products, take the pill only in the morning, or take it on an empty stomach.

A male patient with hepatitis C will begin triple drug therapy with pegylated interferon alfa 2a [Pegasys], ribavirin [Ribasphere], and boceprevir [Victrelis]. The patient tells the nurse that his wife is pregnant. What will the nurse tell him? a.Boceprevir is contraindicated in males whose partners are pregnant. b.He should use a barrier contraceptive when having sex. c.He should use dual drug therapy with pegylated interferon alfa and ribavirin only. d.This combination drug therapy is safe for him to use.

ANS: A The triple combination is dangerous for pregnant women whose partners are using it, so it is contraindicated for any man whose partner is pregnant. Barrier contraceptives should be used by couples to prevent pregnancy when either partner is taking the triple combination therapy. Ribavirin is teratogenic and is not safe when a partner is pregnant. This combination is not safe for pregnant women whose partners are taking these drugs.

A nurse is preparing to administer medications to a patient recently started on delavirdine [Rescriptor]. Which concurrent prescription should the nurse question before administration? a.Alprazolam [Xanax] b.Diphenhydramine [Benadryl] c.Morphine d.Penicillin

ANS: A To prevent toxicity from excessive drug levels, patients should not take alprazolam while taking delavirdine. Diphenhydramine, morphine, and penicillin are not contraindicated for patients taking delavirdine.

A patient who has been receiving intravenous gentamicin for several days reports having had a headache for 2 days. The nurse will request an order to: a. discontinue the gentamicin. b. obtain a gentamicin trough before the next dose is given c. give an analgesic to control headache discomfort. d. obtain renal function tests to evaluate for potential nephrotoxicity.

ANS: A A persistent headache may be a sign of developing ototoxicity, and since ototoxicity is largely irreversible, gentamicin should be withdrawn at the first sign of developing ototoxicity. A gentamicin trough should be obtained before the next dose is given when high gentamicin levels are suspects. Analgesics are not indicated until a serious cause of the headache has been ruled out. A headache is an early sign of ototoxicity, not nephrotoxicity.

2. A 65-year-old patient who receives glucocorticoids for arthritis is admitted to the hospital for treatment of a urinary tract infection. The prescriber has ordered intravenous ciprofloxacin [Cipro]. Before administering the third dose of this drug, the nurse reviews the bacterial culture report and notes that the causative organism is Escherichia coli. The bacterial sensitivity report is pending. The patient complains of right ankle pain. What will the nurse do? a. Withhold the dose of ciprofloxacin and notify the provider of the patient's symptoms. b. Instruct the patient to exercise the right foot and ankle to minimize the pain. c. Question the patient about the consumption of milk and any other dairy products. d. Request an order to increase this patient's dose of glucocorticoids.

ANS: A A rare but serious adverse effect associated with fluoroquinolones is tendon rupture, and those at highest risk are children, patients older than 60 years, transplant patients, and any patients taking glucocorticoids. Any pain in either heel should be reported and the drug should be discontinued. Patients should be instructed not to exercise until tendonitis has been ruled out. Dairy products can reduce the absorption of ciprofloxacin, so this is not a concern with this patient. Because the pain may be caused by tendonitis associated with ciprofloxacin, it is not correct to request an increase in the glucocorticoid dosing.

6. A prescriber has ordered cefoxitin for a patient who has an infection caused by a gram-negative bacteria. The nurse taking the medication history learns that the patient experienced a maculopapular rash when taking amoxicillin [Amoxil] several years earlier. What will the nurse do? a. Administer the cefoxitin and observe for any side effects. b. Give the cefoxitin and have epinephrine and respiratory support available. c. Request an order for a different, nonpenicillin, noncephalosporin antibiotic. d. Request an order to administer a skin test before giving the cefoxitin.

ANS: A Because of structural similarities between penicillins (PCNs) and cephalosporins, a few patients allergic to one drug type will be allergic to the other drug type, although this is rare. For patients with mild PCN allergy, such as rash, cephalosporins can be used with minimal concern, so it is correct to administer the drug and monitor for side effects. It is unnecessary to prepare for anaphylaxis, to give another class of drug, or to administer a skin test.

5. A child with an ear infection is not responding to treatment with amoxicillin [Amoxil]. The nurse will expect the provider to order: a. amoxicillin-clavulanic acid [Augmentin]. b. ampicillin. c. nafcillin. d. penicillin G [Benzylpenicillin].

ANS: A Beta-lactamase inhibitors are drugs that inhibit bacterial beta-lactamases. These drugs are always given in combination with a penicillinase-sensitive penicillin. Augmentin contains amoxicillin and clavulanic acid and is often used when patients fail to respond to amoxicillin alone. Ampicillin is similar to amoxicillin, but amoxicillin is preferred and, if drug resistance occurs, ampicillin is equally ineffective. Pharmaceutical chemists have developed a group of penicillins that are resistant to inactivation by beta-lactamases (eg, nafcillin), but these drugs are indicated only for penicillinase-producing strains of staphylococci. Penicillin G would be as ineffective as amoxicillin if beta-lactamase is present.

6. A patient who is receiving intravenous ciprofloxacin for pneumonia develops diarrhea. A stool culture is positive for Clostridium difficile. The nurse will expect the provider to: a. add metronidazole [Flagyl]. b. increase the dose of ciprofloxacin. c. restrict dairy products. d. switch to gemifloxacin.

ANS: A C. difficile is resistant to fluoroquinolones; metronidazole is the drug of choice to treat this infection. Metronidazole is lethal only to anaerobic organisms, so the ciprofloxacin should be continued to treat the pneumonia. Increasing the dose of ciprofloxacin is not indicated, because C. difficile is resistant to ciprofloxacin. Gemifloxacin is approved for use in respiratory infections.

14. Which side effect of clindamycin [Cleocin] causes the most concern and may warrant discontinuation of the drug? a. Diarrhea b. Headache c. Nausea d. Vomiting

ANS: A CDAD is a serious, sometimes fatal suprainfection associated with clindamycin. Patients with diarrhea should notify their prescriber immediately and discontinue the drug until this condition has been ruled out. Headache, nausea, and vomiting do not warrant discontinuation of the drug and are not associated with severe side effects.

A nurse is explaining to nursing students why a cephalosporin is used in conjunction with an aminoglycoside for a patient with an infection. Which statement by a student indicates understanding of the teaching? a. "Cephalosporins enhance the actions of aminoglycosides by weakening bacterial cell walls." b. "Cephalosporins prevent neuromuscular blockade associated with aminoglycosides." c. "Cephalosporins prolong the postantibiotic effects of the aminoglycosides so doses can be decreased." d. "Cephalosporins reduce bacterial resistance to aminoglycosides."

ANS: A Cephalosporins, penicillins, and vancomycin can be used in conjunction with aminoglycosides; these drugs weaken the bacterial cell wall and enhance the bactericidal actions of aminoglycosides. Cephalosporins do not prevent neuromuscular blockade. They do not prolong the postantibiotic effects of aminoglycosides. They do not affect bacterial resistance.

12. A nurse is providing teaching for a patient who will begin taking clarithromycin ER [Biaxin XL] to treat an Helicobacter pylori infection. Which statement by the patient indicates understanding of the teaching? a. "I may experience distorted taste when taking this medication." b. "I should take 1 tablet twice daily for 10 days." c. "I should take this medication on an empty stomach." d. "This medication does not interact with other drugs."

ANS: A Clarithromycin is available in an extended-relief preparation as Biaxin XL. Biaxin can cause distortion of taste, so patients should be warned of this side effect. Biaxin XL should be taken once and not twice daily. Biaxin should be taken with food. Biaxin interacts with other drugs by inhibiting hepatic metabolism of those drugs.

1. A nurse is preparing to administer an antibiotic to a patient with methicillin-resistant Staphylococcus aureus (MRSA). The nurse would expect the healthcare provider to order which antibiotic? a. Daptomycin [Cubicin] b. Levofloxacin [Levaquin] c. Norfloxacin [Noroxin] d. Ciprofloxacin [Cipro]

ANS: A Daptomycin is active against MRSA. Levofloxacin and norfloxacin are not approved to treat MRSA. Ciprofloxacin is a poor choice for staphylococcal infections, including MRSA.

10. A child with an upper respiratory infection caused by B. pertussis is receiving erythromycin ethylsuccinate. After 2 days of treatment, the parent asks the nurse why the child's symptoms have not improved. Which response by the nurse is correct? a. "Erythromycin eliminates the bacteria that causes the infection, but not the toxin that causes the symptoms." b. "We may need to add penicillin or another antibiotic to increase the antimicrobial spectrum." c. "We will need to review the culture sensitivity information to see whether a different antibiotic is indicated." d. "Your child may have developed a suprainfection that we need to culture and treat."

ANS: A Erythromycin is the drug of first choice for treating pertussis infections. Because symptoms are caused by a bacterial toxin and not by the bacteria itself, the drug eliminates the bacteria but does little to alter the course of the disease. It is given to lower infectivity. It is not necessary to add another antibiotic, review the sensitivity information, or look for a suprainfection.

9. A patient is to undergo orthopedic surgery, and the prescriber will order a cephalosporin to be given preoperatively as prophylaxis against infection. The nurse expects the provider to order which cephalosporin? a. First-generation cephalosporin b. Second-generation cephalosporin c. Third-generation cephalosporin d. Fourth-generation cephalosporin

ANS: A First-generation cephalosporins are widely used for prophylaxis against infection in surgical patients, because they are as effective, less expensive, and have a narrower antimicrobial spectrum than second-, third-, and fourth-generation cephalosporins.

A patient who is taking gentamicin and a cephalosporin for a postoperative infection requests medication for mild postsurgical pain. The nurse will expect to administer which of the following medications? a. Acetaminophen b. Aspirin c. Ibuprofen d. Morphine

ANS: A Gentamicin and cephalosporins are both nephrotoxic. This patient should avoid taking other potentially nephrotoxic drugs. Acetaminophen is not nephrotoxic and may be given for mild pain. Aspirin and ibuprofen are both nephrotoxic. Morphine is not nephrotoxic but is not indicated for mild pain.

1. A nurse transcribes a new prescription for potassium penicillin G given intravenously (IV) every 8 hours and gentamicin given IV every 12 hours. Which is the best schedule for administering these drugs? a. Give the penicillin at 0800, 1600, and 2400; give the gentamicin [Garamycin] at 1800 and 0600. b. Give the penicillin at 0800, 1600, and 2400; give the gentamicin [Garamycin] at 1200 and 2400. c. Give the penicillin at 0600, 1400, and 2200; give the gentamicin [Garamycin] at 0600 and 1800. d. Give the penicillin every 8 hours; give the gentamicin [Garamycin] simultaneously with two of the penicillin doses.

ANS: A Gentamicin should never be administered concurrently with penicillin, because they will interact, and the penicillin may inactivate the aminoglycoside. All the other options show concurrent administration.

A patient is diagnosed with an infection caused by Staphylococcus aureus, and the prescriber orders intravenous gentamicin and penicillin (PCN). Both drugs will be given twice daily. What will the nurse do? a. Administer gentamicin, flush the line, and then give the penicillin. b. Give the gentamicin intravenously and the penicillin intramuscularly. c. Infuse the gentamicin and the penicillin together to prevent fluid overload. d. Request an order to change the penicillin to vancomycin.

ANS: A Gentamicin should not be infused with penicillins in the same solution, because PCN inactivates gentamicin; therefore, the nurse should give one first, flush the line, and then give the other. The nurse cannot give a drug IM when it is ordered IV without an order from the prescriber. These two drugs should not be infused in the same solution. There is no indication for changing the PCN to vancomycin; that should be done for serious infections.

12. A nurse is discussing methicillin-resistant Staphylococcus aureus (MRSA) with a group of nursing students. Which statement by a student correctly identifies the basis for MRSA resistance? a. "MRSA bacteria have developed PBPs with a low affinity for penicillins." b. "MRSA bacteria produce penicillinases that render penicillin ineffective." c. "MRSA occurs because of host resistance to penicillins." d. "MRSA strains replicate faster than other Staphylococcus aureus strains."

ANS: A MRSA strains have a unique mechanism of resistance, which is the production of PBPs with a low affinity for penicillins and all other beta-lactam antibiotics. MRSA resistance is not related to beta-lactamase production. MRSA resistance refers to bacterial and not host resistance. The resistance of MRSA strains is not related to speed of replication.

A patient has a Pseudomonas aeruginosa infection that is sensitive to aminoglycosides, and the prescriber orders gentamicin. The patient tells the nurse that a friend received amikacin [Amikin] for a similar infection and wonders why amikacin was not ordered. What will the nurse tell the patient? a. "Amikacin is given when infectious agents are resistant to other aminoglycosides." b. "Amikacin is more vulnerable to inactivation by bacterial enzymes." c. "Amikacin is a narrow-spectrum drug and will probably not work for this infection." d. "Gentamicin is less toxic to the ears and the kidneys."

ANS: A Resistance to amikacin is uncommon at this point; to minimize the emergence of amikacin- resistant bacteria, this drug is reserved for infections in which resistance to other aminoglycosides has developed. Amikacin is the least susceptible to inactivation by bacterial enzymes. Amikacin is a broad-spectrum antibiotic. All aminoglycosides are ototoxic and nephrotoxic.

2. A pregnant adolescent patient asks the nurse whether she should continue to take her prescription for tetracycline [Sumycin] to clear up her acne. Which response by the nurse is correct? a. "Tetracycline can be harmful to the baby's teeth and should be avoided." b. "Tetracycline is safe to take during pregnancy." c. "Tetracycline may cause allergic reactions in pregnant women." d. "Tetracycline will prevent asymptomatic urinary tract infections."

ANS: A Tetracyclines can cause discoloration of deciduous teeth of infants if taken by the mother after the fourth month of gestation. Tetracyclines should not be given to pregnant women. Tooth discoloration can be prevented if the drugs are not taken by pregnant women or by children under 8 years of age. Tetracycline is not appropriate for a pregnant patient. Pregnancy does not precipitate an allergic response to tetracycline. Tetracycline should not be used to prevent urinary tract infections (UTIs), especially in pregnant women.

1. A nurse is providing education about tetracycline [Sumycin]. Which statement by the patient best demonstrates understanding of the administration of this medication? a. "I should not take this medication with milk or other dairy products." b. "I should not worry if I experience an acnelike rash with this medication." c. "I should take an antacid, such as Tums, if I experience gastrointestinal distress." d. "I should take this antibiotic with a calcium supplement to improve absorption."

ANS: A The patient should avoid taking the medication with dairy products to help prevent chelation. An acnelike reaction would indicate an allergic response. Taking the medication with calcium-containing antacids or supplements should be avoided, because this also leads to chelation.

A patient is receiving tobramycin three times daily. A tobramycin peak level is 4.5 and the trough is 1.2. What will the nurse do? a. Give the next dose as ordered. b. Hold the next dose and notify the provider. c. Monitor the patient for signs of nephrotoxicity. d. Tell the patient to report tinnitus.

ANS: A These levels are within normal limits, so the next dose may be given safely. It is not necessary to withhold the next dose. These levels do not indicate any increased risk of nephrotoxicity or ototoxicity.

8. A patient with no known drug allergies is receiving amoxicillin [Amoxil] PO twice daily. Twenty minutes after being given a dose, the patient complains of shortness of breath. The patient's blood pressure is 100/58 mm Hg. What will the nurse do? a. Contact the provider and prepare to administer epinephrine. b. Notify the provider if the patient develops a rash. c. Request an order for a skin test to evaluate possible PCN allergy. d. Withhold the next dose until symptoms subside.

ANS: A This patient is showing signs of an immediate penicillin allergy, that is, one that occurs within 2 to 30 minutes after administration of the drug. The patient is showing signs of anaphylaxis, which include laryngeal edema, bronchoconstriction, and hypotension; these must be treated with epinephrine. This is an emergency, and the provider must be notified immediately, not when other symptoms develop. It is not necessary to order skin testing. The patient must be treated immediately, and subsequent doses should not be given.

7. A patient will be discharged home to complete treatment with intravenous cefotetan with the assistance of a home nurse. The home care nurse will include which instruction when teaching the patient about this drug treatment? a. Abstain from alcohol consumption during therapy. b. Avoid dairy products while taking this drug. c. Take an antihistamine if a rash occurs. d. Use nonsteroidal anti-inflammatory drugs (NSAIDs), not acetaminophen, for pain.

ANS: A Two cephalosporins, including cefotetan, can induce a state of alcohol intolerance and cause a disulfiram-like reaction when alcohol is consumed; therefore, patients should be advised to avoid alcohol. It is not necessary to avoid dairy products. Patients who experience a rash should report this to their provider. Cefotetan can also promote bleeding, so drugs that inhibit platelet aggregation should be avoided.

5. A provider has ordered ceftriaxone 4 gm once daily for a patient with renal impairment. What will the nurse do? a. Administer the medication as prescribed. b. Contact the provider to ask about giving the drug in divided doses. c. Discuss increasing the interval between doses with the provider. d. Discuss reducing the dose with the provider.

ANS: A Unlike other cephalosporins, ceftriaxone is eliminated largely by the liver, so dosage reduction is unnecessary in patients with renal impairment. Giving the drug in divided doses, increasing the interval between doses, and reducing the dose are not necessary.

A patient is receiving gentamicin once daily. A nursing student asks the nurse how the drug can be effective if given only once a day. The nurse explains drug dosing schedules for aminoglycosides. Which statement by the student indicates a need for further teaching? a. "Gentamicin has a longer half-life than other aminoglycosides." b. "Large doses given once daily yield higher peak levels." c. "The postantibiotic effect lasts for several hours." d. "There is less risk of ototoxicity and nephrotoxicity with large daily doses."

ANS: A When a daily dose is given once daily instead of divided into 2 or 3 doses, a higher peak level can be achieved. The higher peak, along with the fact that aminoglycosides have a postantibiotic effect, means that the bacterial kill is just as great with one dose as with 2 or 3 doses per day. When a single daily dose is given, the risk of toxicity is reduced. Gentamicin does not have a longer half-life than other aminoglycosides.

A patient is admitted to the unit for treatment for an infection. The patient receives IV amikacin [Amikin] twice a day. When planning for obtaining a peak aminoglycoside level, when should the nurse see that the blood is drawn? a. 30 minutes after the IV infusion is complete b. 1 hour after the IV infusion is complete c. 1 hour before administration of the IV infusion d. A peak level is not indicated with twice-daily dosing.

ANS: A When divided daily doses are used, blood samples for measurement of peak levels are drawn 1 hour after IM injection and 30 minutes after completion of an IV infusion. This medication is administered IV, so blood draws must follow 30 minutes after infusion to obtain peak levels. Measurement of peak levels is unnecessary only when a single daily dose is used.

2. A patient has an infection caused by Pseudomonas aeruginosa. The prescriber has ordered piperacillin and amikacin, both to be given intravenously. What will the nurse do? a. Make sure to administer the drugs at different times using different IV tubing. b. Suggest giving larger doses of piperacillin and discontinuing the amikacin. c. Suggest that a fixed-dose combination of piperacillin and tazobactam [Zosyn] be used. d. Watch the patient closely for allergic reactions, because this risk is increased with this combination.

ANS: A When penicillins are present in high concentrations, they interact with aminoglycosides and inactivate the aminoglycoside; therefore, these two drugs should never be mixed in the same IV solution. The drugs should be given at different times with different tubing. In the treatment of Pseudomonas infections, extended-spectrum penicillins, such as piperacillin, usually are given in conjunction with an antipseudomonal aminoglycoside, such as amikacin; therefore, suggesting a larger dose of piperacillin and discontinuation of the amikacin is incorrect. Zosyn is not recommended. The risk of allergic reactions does not increase with this combination of drugs.

A patient is diagnosed with a lung infection caused by P. aeruginosa. The culture and sensitivity report shows sensitivity to all aminoglycosides. The nurse knows that the rate of resistance to gentamicin is common in this hospital. The nurse will expect the provider to order which medication? a. Amikacin [Amikin] b. Gentamicin c. Paromomycin d. Tobramycin

ANS: A When resistance to gentamicin and tobramycin is common, amikacin is the drug of choice for initial treatment of aminoglycoside-sensitive infections. Gentamicin would not be indicated, because resistance is more likely to develop. Paromomycin is used only for local effects within the intestine and is given orally. Tobramycin is not indicated, because organisms can more readily develop resistance.

A patient complains of painful urination. A physical examination reveals vesicles on her labia, vagina, and the foreskin of her clitoris. The nurse will expect to teach this patient about which medication? a. Acyclovir (Zovirax) b. Azithromycin (Zithromax) c. Metronidazole (Flagyl) d. Tinidazole (Tindamax)

ANS: A Genital herpes can be treated with acyclovir, famciclovir, or valacyclovir, which are antiviral medications. Azithromycin, metronidazole, and tinidazole are antibiotics and do not have antiviral effects.

A patient has come to the STD clinic and has been diagnosed with a Trichomonas vaginalisinfection. What education should be provided to this patient? a. Male partners should always be treated, even if they are asymptomatic. b. The applicator for the vaginal gel should be washed after each application. c. The infection is not completely eliminated with the medication. d. The medication should be taken twice daily for 2 weeks.

ANS: A The most important information the nurse can provide is that male partners should always be treated, even if they are asymptomatic. Gels are not indicated for T. vaginalis; oral medications are. T. vaginalis infection can be easily treated with a single dose of metronidazole or tinidazole. Both medications are given as a single dose.

During a routine screening, an asymptomatic, pregnant patient at 37 weeks' gestation learns thatshe has an infection caused by Chlamydia trachomatis. The nurse will expect the provider to order which drug? a.Azithromycin b.Doxycycline c.Erythromycin ethylsuccinate d.Sulfisoxazole

ANS: A The preferred treatment for C. trachomatis during pregnancy is either azithromycin or amoxicillin. Doxycycline can be used for nonpregnant patients. Erythromycin is used for infants. Sulfisoxazole is not recommended for pregnant woman near term, because it can cause kernicterus in the infant.

An adolescent patient comes to the clinic complaining of a burning sensation upon urination and a pus-like discharge from the penis. The nurse is correct to suspect that the patient has which disorder? a. Gonorrhea b.Herpes simplex c. Nongonococcal urethritis d.Syphilis

ANS: A This patient has signs and symptoms consistent with gonorrhea. Herpes simplex manifests as vesicles on the penis or testes (or both) with a watery discharge. Nongonococcal urethritis results in ulcers on the urethral site. Syphilis is characterized by chancres and skin lesions.

A nursing student wants to know the differences between hospital-associated methicillin-resistant Staphylococcus aureus (HA-MRSA) and community-associated methicillin-resistant Staphylococcus aureus (CA-MRSA). Which statements about CA-MRSA are true? (Select all that apply.) a. 20% to 30% of the general population are colonized with CA-MRSA. b. Boils caused by CA-MRSA can be treated without antibiotics. c. CA-MRSA is less dangerous than HA-MRSA. d. CA-MRSA does not cause necrotizing fasciitis. e. CA-MRSA is transmitted by airborne droplets.

ANS: A, B, C CA-MRSA is thought to be present in 20% to 30% of the population, and many of these individuals are asymptomatic carriers. Boils caused by CA-MRSA can often be treated by surgical drainage alone. CA-MRSA is less dangerous than HA-MRSA but more dangerous than methicillin-sensitive Staphylococcus aureus (MSSA). CA-MRSA generally causes mild skin infections but can cause more serious infections, such as necrotizing fasciitis. CA-MRSA is transmitted by skin-to-skin contact and by contact with contaminated objects.

Besides the cost of administering a given drug, which are considerations when a provider selects a cephalosporin to treat an infection? (Select all that apply.) a. Adverse effects b. Antimicrobial spectrum c. Brand name d. Manufacturer e. Pharmacokinetics

ANS: A, B, E Cephalosporins tend to be more alike than different, but some differences make some preferable to others. Adverse effects, antimicrobial spectrum, and drug pharmacokinetics affect a prescriber's decision to select one drug over another. The agent's brand name and manufacturer should not have anything to do with drug selection.

The nurse is providing education to a group of patients who are HIV positive. The nurse is discussing the various medications used to treat HIV infection. A patient asks about nevirapine [Viramune]. Which statements by the nurse most accurately reflect the facts about nevirapine? (Select all that apply.) a."Some herbal preparations can reduce the levels of this drug." b."This agent can damage the liver; therefore, liver function tests are needed periodically." c."Usually no adverse effects occur when this medication is used alone." d."The drug must be dosed five times per day at evenly spaced intervals." e."You should call your healthcare provider immediately if you develop a rash."

ANS: A, B, E Some herbal preparations can reduce the levels of nevirapine, which can damage the liver. The patient should call the healthcare provider immediately if a rash develops. Many adverse effects are associated with the use of nevirapine. In adults, nevirapine is dosed once daily initially, not five times a day.

Which infection(s) may be treated with linezolid [Zyvox])? (Select all that apply.) a. Community-acquired pneumonia (CAP) that is penicillin sensitive b. Nosocomial pneumonia caused by methicillin-sensitive Staphylococcus aureus (MSSA) c. Pneumonias caused by Mycoplasma avium d. Superficial methicillin-resistant Staphylococcus aureus skin infections (MRSA) e. Vancomycin-resistant infections

ANS: A, B, E Linezolid is indicated for CAP caused by PCN-sensitive strains of Streptococcus pneumoniae, nosocomial pneumonia caused by MSSA and MRSA, and vancomycin-resistant enterococcal (VRE) infections. It is not recommended for M. avium infections or for superficial skin infections caused by MRSA.

Tetracyclines are considered first-line drugs for which disorder(s)? (Select all that apply.) a. Chlamydia trachomatis cervicitis b. Clostridium difficile diarrhea c. Lyme disease d. Methicillin-resistant Staphylococus aureus skin infections e. Typhus fever

ANS: A, C, E Tetracyclines are drugs of first choice for rickettsial diseases, infections caused by C. trachomatis, brucellosis, cholera, Mycoplasma pneumonia, Lyme disease, anthrax, and gastric infections caused by H. pylori. They are not first-line drugs for CDAD or MRSA skin infections.

Which antibiotics may be administered topically? (Select all that apply.) a. Bacitracin b. Daptomycin c. Ofloxacin d. Polymyxin B e. Rifampin

ANS: A, D Bacitracin and Polymyxin B are both topical antibiotics. Daptomycin, ofloxacin, and rifampin are not formulated to be administered topically.

A patient with hepatitis B begins treatment with adefovir [Hepsera] and asks the nurse how long the drug therapy will last. The nurse will tell the patient that the medication will need to be taken for: a.a lifetime. b.an indefinite, prolonged period of time. c.48 weeks. d.until nephrotoxicity occurs

ANS: B Current guidelines recommend treatment only for patients at highest risk; it is unknown whether treatment should continue lifelong. Treatment is usually prolonged, without a specific period of time. Nephrotoxicity is common but is not the deciding factor when determining length of effective treatment.

A patient has a positive test for hepatitis C and is admitted to the hospital. The admission laboratory tests reveal a normal ALT, and a liver biopsy is negative for hepatic fibrosis and inflammation. The nurse will prepare this patient for: a.dual therapy with pegylated interferon alfa and ribavirin. b.no medication therapy at this time. c.pegylated interferon alfa only until ALT levels are elevated. d.triple drug therapy with pegylated interferon alfa, ribavirin, and boceprevir.

ANS: B Current recommendations are that treatment is used only for patients with HCV viremia, persistent elevation of ALT, and evidence of hepatic fibrosis and inflammation upon liver biopsy. Dual therapy has been the regimen of choice for patients with the above symptoms, but the addition of a protease inhibitor has been shown to improve outcomes. It is not correct to give pegylated interferon alfa until ALT levels are elevated. Triple drug therapy is used for patients with the above symptoms.

A patient with HIV contracts herpes simplex virus (HSV), and the prescriber orders acyclovir [Zovirax] 400 mg PO twice daily for 10 days. After 7 days of therapy, the patient reports having an increased number of lesions. The nurse will expect the provider to: a.extend this patient's drug therapy to twice daily for 12 months. b.give intravenous foscarnet every 8 hours for 2 to 3 weeks. c.increase the acyclovir dose to 800 mg PO 5 times daily. d.order intravenous valacyclovir [Valtrex] 1 gm PO twice daily for 10 days

ANS: B Foscarnet is active against all known herpesviruses and is used in immunocompromised patients with acyclovir-resistant HSV or VZV. This patient is demonstrating resistance to acyclovir, so extending acyclovir therapy or increasing the acyclovir dose will not be effective. Valacyclovir is not approved for use in immunocompromised patients because of the risk for thrombotic thrombocytopenic purpura/hemolytic uremic syndrome

A nurse is teaching a group of nursing students about influenza prevention. Which statement by a student indicates understanding of the teaching? a."I may develop a mild case of influenza if I receive the vaccine by injection." b."I should receive the vaccine every year in October or November." c."If I have a cold I should postpone getting the vaccine." d."The antiviral medications are as effective as the flu vaccine for preventing the flu."

ANS: B Influenza vaccine should be given every year in October or November. The vaccine will not cause influenza. Minor illnesses, such as a cold, are not a contraindication for receiving the vaccine. Antiviral medications are not as effective as the flu vaccine in preventing influenza.

A patient has a positive test for influenza type A and tells the nurse that symptoms began 5 days before being tested. The prescriber has ordered oseltamivir [Tamiflu]. The nurse will tell the patient that oseltamivir: a.may decrease symptom duration by 2 or 3 days. b.may not be effective because of the delay in starting treatment. c.may reduce the severity but not the duration of symptoms. d.will alleviate symptoms within 24 hours of the start of therapy.

ANS: B Oseltamivir is most effective when begun within 2 days after symptom onset. When started within 12 hours of symptom onset, it may decrease duration of symptoms by 2 to 3 days. The drug reduces both symptom severity and symptom duration when used in a timely fashion. It does not rapidly alleviate symptoms.

The nurse is preparing to discharge a patient with HIV who will continue to take enfuvirtide [Fuzeon] at home. The nurse is providing patient education about the medication. What information about the administration of enfuvirtide is most appropriate for the patient? a.The importance of injecting the drug into two alternating sites daily b.How to reconstitute and self-administer a subcutaneous injection c.The importance of taking the drug with high doses of vitamin E d.Likely drug interactions between enfuvirtide and other antiretroviral drugs

ANS: B The most appropriate information about the administration of enfuvirtide is how to reconstitute and self-administer a subcutaneous injection. The medication should never be injected into the same site or just between two sites. Vitamin E is not indicated for this medication. Enfuvirtide does not appear to cause significant drug interactions.

A patient taking stavudine [Zerit] telephones the clinic and reports numbness and tingling in the hands and feet. What should the nurse tell the patient? a.The numbness is an expected side effect of the medication and will diminish once the drug is withdrawn. b.The medication will probably be stopped, and the patient should come into the clinic for further evaluation. c.The dose may be too high, and the patient should cut the tablet in half. d.The patient should take the medication on a full stomach to reduce absorption of the drug.

ANS: B The patient has early signs and symptoms of neuropathy, which may resolve if the drug is stopped. The patient should be taught early in treatment to report these symptoms immediately. Numbness is not an expected side effect and these symptoms may diminish once the drug is withdrawn. The patient should never be advised to cut the dose in half unless instructed to do so by a prescriber. Taking the medication on a full stomach will not affect the amount of medication absorbed.

The nurse is performing a physical assessment on a patient who is receiving treatment with abacavir, zidovudine, and lamivudine [Trizivir]. The patient complains of fatigue. Upon further assessment, the nurse finds a rash and notes that the patient has a temperature of 101.1°F. What is the nurse's best course of action? a.Tell the patient that this is an expected response to these medications and to continue the agents as prescribed. b.Have the patient hold the medications and arrange for an immediate evaluation by the prescriber. c.Have the patient continue the abacavir but discontinue the other two agents for 3 weeks. d.Instruct the patient to continue all three medications and administer an antihistamine for the symptoms.

ANS: B The patient should discontinue all the medications. Immediate assessment by the provider is required, because the patient is showing early symptoms of a fatal hypersensitivity reaction. This is not an expected response; it indicates a serious reaction, which the patient should report to the prescriber immediately. The patient should not continue the medications for any additional dosages.

A patient is receiving an intraperitoneal aminoglycoside during surgery. To reverse a serious side effect of this drug, the nurse may expect to administer which agent? a. Amphotericin B b. Calcium gluconate c. Neuromuscular blocker d. Vancomycin

ANS: B Aminoglycosides can inhibit neuromuscular transmission, especially during intraperitoneal or intrapleural instillation, and this risk is increased when neuromuscular blocking agents and general anesthetics are given. Calcium can reverse neuromuscular blockade. Amphotericin B, additional neuromuscular blockers, and vancomycin are not indicated.

3. A patient will receive oral ciprofloxacin [Cipro] to treat a urinary tract infection. The nurse provides teaching for this patient. Which statement by the patient indicates a need for further teaching? a. "I may have abdominal pain and nausea, but these are usually mild." b. "I should take this medication with food or milk to improve absorption." c. "I should stop taking the medication immediately if I experience heel pain." d. "I will need to use sunscreen every time I go outdoors."

ANS: B Dairy products inhibit the absorption of ciprofloxacin, so they should be avoided. Abdominal pain and nausea and vomiting are common and usually mild. Patients should stop taking the drug if heel pain occurs until tendonitis has been ruled out. Photosensitivity can occur, so sunscreen should be used.

A patient who has been taking gentamicin for 5 days reports a headache and dizziness. What will the nurse do? a. Request an order for a gentamicin peak level. b. Suspect ototoxicity and notify the prescriber. c. Tell the patient to ask for help with ambulation. d. Tell the patient to report any tinnitus.

ANS: B Headache and dizziness are signs of ototoxicity, and the prescriber should be notified. A peak level is not indicated; it is more important to know the trough level. Telling the patient to ask for help with ambulation and to report tinnitus should both be done but neither one is the priority nursing action.

11. A patient is taking erythromycin ethylsuccinate for a chlamydial infection and develops vaginal candidiasis. The prescriber orders ketoconazole to treat the superinfection. What will the nurse do? a. Administer the erythromycin and the ketoconazole as ordered. b. Contact the provider to discuss changing to a different antifungal medication. c. Contact the provider to discuss increasing the dose of erythromycin. d. Contact the provider to suggest using erythromycin stearate.

ANS: B Erythromycin can prolong the QT interval when present in large concentrations. When erythromycin is combined with a CYP3A4 inhibitor, such as ketoconazole, the risk of sudden cardiac death increases fivefold. The nurse should discuss changing the antifungal medication to one that is not a CYP3A4 inhibitor. It is not correct to give the ketoconazole without questioning the order. Increasing the dose of erythromycin would increase the risk of QT prolongation. Changing to a different preparation of erythromycin would not alter the risk.

A patient who takes the loop diuretic ethacrynic acid is given intravenous gentamicin for an infection. After several days of treatment with gentamicin, the nurse reviews the patient's most recent laboratory results and notes a gentamicin trough of 2.1 mcg/mL and normal blood urea nitrogen (BUN) and serum creatinine levels. The nurse will question the patient about: a. gastrointestinal (GI) symptoms. b. headache, dizziness, or vertigo. c. presence of rash. d. urine output.

ANS: B Ethacrynic acid has ototoxic properties, and patients who take this drug with an aminoglycoside have an increased risk of ototoxicity, especially when trough levels of the aminoglycoside are elevated. A trough level of 2.1 mcg/mL is above normal limits for gentamicin, so this patient should be asked about early signs of ototoxicity. There is no indication to evaluate for GI symptoms, rash, or urine output.

3. A woman complains of burning on urination and increased frequency. The patient has a history of frequent urinary tract infections (UTIs) and is going out of town in 2 days. To treat the infection quickly, the nurse would expect the healthcare provider to order: a. aztreonam [Azactam]. b. fosfomycin [Monurol]. c. trimethoprim/sulfamethoxazole [Bactrim]. d. vancomycin [Vancocin].

ANS: B Fosfomycin has been approved for single-dose therapy of UTIs in women. Vancomycin and aztreonam are not indicated for UTIs. Bactrim is indicated for UTIs, but administration of a single dose is not therapeutic.

A nurse is preparing to administer a dose of gentamicin to a patient who is receiving the drug 3 times daily. The nurse will monitor ____ levels. a. peak b. peak and trough c. serum drug d. trough

ANS: B When divided doses of aminoglycosides are given, it is important to measure both peak and trough levels of the drug, because it is more difficult to achieve therapeutic peaks in lower doses without causing toxicity. Trough levels are drawn when single-dosing regimens are used, because high peak levels are guaranteed.

3. A nurse assisting a nursing student with medications asks the student to describe how penicillins (PCNs) work to treat bacterial infections. The student is correct in responding that penicillins: a. disinhibit transpeptidases. b. disrupt bacterial cell wall synthesis. c. inhibit autolysins. d. inhibit host cell wall function.

ANS: B PCNs weaken the cell wall, causing bacteria to take up excessive amounts of water and subsequently rupture. PCNs inhibit transpeptidases and disinhibit autolysins. PCNs do not affect the cell walls of the host.

17. A nurse is teaching a nursing student about dalfopristin/quinupristin [Synercid]. Which statement by the student indicates an understanding of the teaching? a. "Patients should stop taking the drug if they experience joint and muscle pain." b. "Patients taking this drug should have blood tests performed frequently." c. "Patients who are allergic to penicillin should not take this drug." d. "This drug will be administered intravenously over a 30- to 60-minute period."

ANS: B Patients taking dalfopristin/quinupristin should have blood levels measured twice the first week and then weekly thereafter to assess for hepatotoxicity. Joint and muscle pain are not an indication for withdrawing the drug. There is no cross-sensitivity to penicillin. The drug is given intravenously over a period of at least 1 hour.

10. A patient receiving a cephalosporin develops a secondary intestinal infection caused by Clostridium difficile. What is an appropriate treatment for this patient? a. Adding an antibiotic, such as vancomycin [Vancocin], to the patient's regimen b. Discontinuing the cephalosporin and beginning metronidazole [Flagyl] c. Discontinuing all antibiotics and providing fluid replacement d. Increasing the dose of the cephalosporin and providing isolation measures

ANS: B Patients who develop C. difficile infection (CDI) as a result of taking cephalosporins or other antibiotics need to stop taking the antibiotic in question and begin taking either metronidazole or vancomycin. Adding one of these antibiotics without withdrawing the cephalosporin is not indicated. CDI must be treated with an appropriate antibiotic, so stopping all antibiotics is incorrect. Increasing the cephalosporin dose would only aggravate the CDI.

8. The nurse is caring for a patient who is receiving vancomycin [Vancocin]. The nurse notes that the patient is experiencing flushing, rash, pruritus, and urticaria. The patient's heart rate is 120 beats per minute, and the blood pressure is 92/57 mm Hg. The nurse understands that these findings are consistent with: a. allergic reaction. b. red man syndrome. c. rhabdomyolysis. d. Stevens-Johnson syndrome.

ANS: B Rapid infusion of vancomycin can cause flushing, rash, pruritus, urticaria, tachycardia, and hypotension, a collection of symptoms known as red man syndrome. Rhabdomyolysis is not associated with the administration of vancomycin. The patient's symptoms may seem to indicate an allergic reaction, but this is specifically red man syndrome. The symptoms are not those of Stevens-Johnson syndrome, which manifests as blisters or sores (or both) on the lips and mucous membranes after exposure to the sun.

19. A patient with severe community-acquired pneumonia has been prescribed telithromycin [Ketek]. Which aspect of the patient's medical history is of concern to the nurse? a. Anemia b. Myasthenia gravis c. Renal disease d. Strep. pneumoniae infection

ANS: B Telithromycin is a macrolide antibiotic used only for CAP. Patients with myasthenia gravis may experience rapid muscle weakness after taking the drug, and some have died from respiratory failure, so patients with MG should not take this drug. This drug does not have significant myelosuppression, so anemia is not a concern. The drug causes liver injury, so liver disease, and not renal disease, is a concern. Telithromycin is indicated for treatment of S. pneumonia.

An adolescent patient with mild cervicitis is diagnosed with gonorrhea. The nurse will expect the provider to order which drug(s)? a. Azithromycin (Zithromax), 1 gm PO once, and doxycycline (Vibramycin), 100 mg PO twice daily for 7 days b. Cefixime (Suprax), 400 mg PO once, and azithromycin (Zithromax), 1 gm PO once c. Ceftriaxone (Rocephin), 125 mg IM once d. Doxycycline (Vibramycin), 100 mg IV twice daily for 12 days

ANS: B The only options for treating cervical infection with gonorrhea are cefixime or ceftriaxone. Because a high percentage of patients with gonorrhea also have chlamydial infections, they should be treated with either doxycycline or azithromycin until a chlamydial infection has been ruled out. Azithromycin combined with doxycycline would not treat gonorrhea. Ceftriaxone would treat gonorrhea only. Doxycycline would treat chlamydia only.

4. A patient is diagnosed with periodontal disease, and the provider orders oral doxycycline [Periostat]. The patient asks the purpose of the drug. What is the nurse's response? a. "It is used because of its anti-inflammatory effects." b. "It inhibits collagenase to protect connective tissue in the gums." c. "It reduces bleeding and the pocket depth of oral lesions." d. "It suppresses bacterial growth in the oral mucosa."

ANS: B Two tetracyclines are used for periodontal disease. Doxycycline inhibits collagenase, which destroys connective tissue in the gums. It is not used for anti-inflammatory effects. Minocycline is used to reduce bleeding and pocket depth and to inhibit bacterial growth.

4. A nurse is teaching a nursing student what is meant by "generations" of cephalosporins. Which statement by the student indicates understanding of the teaching? a. "Cephalosporins are assigned to generations based on their relative costs to administer." b. "Cephalosporins have increased activity against gram-negative bacteria with each generation." c. "First-generation cephalosporins have better penetration of the cerebrospinal fluid." d. "Later generations of cephalosporins have lower resistance to destruction by beta-lactamases."

ANS: B With each progression from first-generation agents to fifth-generation agents, the cephalosporins show increased activity against gram-negative organisms, increased resistance to destruction by beta-lactamases, and increased ability to reach the CSF. Cost is not a definitive factor. First-generation drugs have less penetration of the CSF. Resistance to destruction by beta-lactamases increases with increasing generations.

A patient admitted to the hospital is using metronidazole (Flagyl) 0.75% gel. The nurse understands that this agent is used to treat which condition? a. Chlamydia trachomatis b. Gardnerella vaginalis c. Haemophilus ducreyi d. Trichomonas vaginalis

ANS: B Gardnerella vaginalis causes bacterial vaginosis and is treated with metronidazole gel. C. trachomatis is treated with systemic antibiotics. H. ducreyi, which causes chancroid, is treated with systemic antibiotics. T. vaginalis is treated with oral metronidazole.

A nursing student is caring for a patient with community-acquired pneumonia (CAP) who also has a methicillin-resistant Staphylococcus aureus (MRSA) skin infection. The prescriber has ordered daptomycin [Cubicin]. The nurse encourages the student to approach the provider to request a different antibiotic because of which facts about daptomycin? (Select all that apply.) a. It causes significant renal impairment. b. It is approved only for bloodstream and skin infections. c. It increases the risk of serious cardiorespiratory events. d. It is more likely to produce resistant strains of bacteria. e. It is not effective against MRSA infections.

ANS: B, C Daptomycin has been approved only for bloodstream and skin infections caused by S. aureus, including MRSA infections, and would not be indicated for a CAP infection. In patients with CAP, daptomycin poses a higher risk of serious cardiorespiratory side effects. Daptomycin does not cause renal impairment and is no more likely to produce bacterial resistance than other antibiotics. Daptomycin can be used to treat MRSA infections.

The nurse asks a graduate nurse, "When a patient in the initial phase of HIV infection is assessed, which findings would you expect to see?" The graduate nurse would be correct to respond with which conditions? (Select all that apply.) a.Respiratory distress b.Fever c.Myalgia d.Lymphadenopathy e.Insomnia

ANS: B, C, D Fever, myalgia, and lymphadenopathy are early signs associated with HIV infection. Respiratory distress and insomnia are not consistent findings in the initial phase of HIV infection.

Which organisms can be treated with penicillin G (Benzylpenicillin)? (Select all that apply.) a. Methicillin-resistant Staphylococcus aureus b. Neisseria meningitidis c. Pseudomonas aeruginosa d. Streptococcus pyogenes e. Treponema pallidum

ANS: B, D, E Penicillin G is the first drug of choice for N. meningitidis. It is a drug of first choice for infections caused by sensitive gram-positive cocci, including S. pyogenes. It is a drug of choice for T. pallidum. It is not effective against methicillin-resistant S. aureus or P. aeruginosa.

The nurse is caring for a patient who is human immunodeficiency virus (HIV) positive and is taking high doses of zidovudine [Retrovir]. The nurse is providing patient education about the adverse effects of the medication. Which statement by the patient demonstrates a need for further teaching? a."I may experience fatigue from anemia." b."I may be more susceptible to infection from neutropenia." c."I may have a deficiency of vitamin B6." d."I may have a deficiency of folic acid."

ANS: C A deficiency of vitamin B12, not vitamin B6, would be expected; this statement indicates that further teaching is required. With high-dose zidovudine, the patient can expect anemia, neutropenia, and folic acid deficiency.

The nurse is caring for a patient who is taking a protease inhibitor (PI). Upon review of the laboratory test results, the nurse notes that the patient has newly elevated plasma triglycerides and cholesterol. The nurse expects that the prescriber will manage these levels with: a.lovastatin [Mevacor]. b.simvastatin [Zocor]. c.modified diet and exercise. d.Pancrease.

ANS: C All PIs can elevate plasma levels of cholesterol and triglycerides. Potential interventions for hyperlipidemia include modified diet, exercise, and lipid-lowering agents. Lovastatin and simvastatin should be avoided, because they can accumulate to dangerous levels. Pancrease is not indicated to lower triglycerides and cholesterol.

A nurse provides teaching for a patient with cytomegalovirus (CMV) retinitis who will receive the ganciclovir ocular implant [Vitrasert]. Which statement by the patient indicates a need for further teaching? a."My vision may be blurred for 2 to 4 weeks after receiving the implant." b."Surgical placement of the implant is an outpatient procedure." c."The implant will remain in place permanently." d."The implant will slow progression of CMV retinitis."

ANS: C Ganciclovir ocular implants must be replaced every 5 to 8 months and do not remain in place permanently. It is correct that vision may be blurred for 2 to 4 weeks after placement of the implant, that placement is an outpatient procedure, and that the implant will slow progression of CMV retinitis.

A patient who is pregnant has a history of recurrent genital herpesvirus (HSV). The patient asks the nurse what will be done to suppress an outbreak when she is near term. The nurse will tell the patient that: a.antiviral medications are not safe during pregnancy. b.intravenous antiviral agents will be used if an outbreak occurs. c.oral acyclovir [Zovirax] may be used during pregnancy. d.topical acyclovir [Zovirax] must be used to control outbreaks.

ANS: C Oral acyclovir is devoid of serious adverse effects and may be used safely during pregnancy. It is incorrect to tell this patient that antiviral medications are not safe during pregnancy. Oral acyclovir is used to suppress recurrent genital herpes near term; intravenous antiviral medications are not indicated. It is not necessary to rely on topical medications because oral acyclovir is safe.

A nursing student asks a nurse why pegylated interferon alfa is used instead of regular interferon for a patient with hepatitis C. The nurse will tell the student that pegylated interferon: a.decreases the need for additional medications. b.has fewer adverse effects than interferon. c.is administered less frequently than interferon. d.may be given orally to increase ease of use.

ANS: C Pegylated interferon alfa preparations are preferred because of their convenience and superior efficacy. These preparations may be given once weekly instead of three or more times per week like the regular interferon. Using pegylated interferons does not decrease the need for additional medications. Pegylated interferons have similar adverse effects. Pegylated interferons are not given orally.

A patient with second-degree burns is treated with silver sulfadiazine [Silvadene]. A nursing student asks the nurse about the differences between silver sulfadiazine and mafenide [Sulfamylon], because the two are similar products, and both contain sulfonamides. What does the nurse tell the student about silver sulfadiazine? a. It causes increased pain when the medication is applied. b. It has a broader spectrum of antimicrobial sensitivity. c. It has antibacterial effects related to release of free silver. d. It suppresses renal excretion of acid, causing acidosis.

ANS: C Silver sulfadiazine has antibacterial effects primarily related to the release of free silver and not to the sulfonamide portion of the molecule. Unlike mafenide, silver sulfadiazine reduces pain when applied. Silver sulfadiazine and mafenide have similar antibacterial effects. Because silver sulfadiazine does not suppress renal excretion of acid, it has fewer systemic effects.

A patient who is taking immunosuppressant medications develops a urinary tract infection. The causative organism is sensitive to sulfonamides and to another, more expensive antibiotic. The prescriber orders the more expensive antibiotic. The nursing student assigned to this patient asks the nurse why the more expensive antibiotic is being used. Which response by the nurse is correct? a. "Immunosuppressed patients are folate deficient." b. "Patients who are immunosuppressed are more likely to develop resistance." c. "Sulfonamides are bacteriostatic and depend on host immunity to work." d. "Sulfonamides intensify the effects of immunosuppression."

ANS: C Sulfonamides are usually bacteriostatic and require intact host defenses for complete elimination of infection. Immunosuppressed patients are not necessarily folate deficient. There is no increased likelihood of developing bacterial resistance in immunosuppressed patients. Sulfonamides do not affect immunosuppression.

A patient with type 2 diabetes mellitus takes glipizide. The patient develops a urinary tract infection, and the prescriber orders TMP/SMZ. What will the nurse tell the patient? a. Patients with diabetes have an increased risk of an allergic reaction. b. Patients taking TMP/SMZ may need increased doses of glipizide. c. The patient should check the blood glucose level more often while taking TMP/SMZ. d. The patient should stop taking the glipizide while taking the TMP/SMZ.

ANS: C Sulfonamides can intensify the effects of some drugs, including glipizide, which is a sulfonylurea-type hypoglycemic medication. These drugs may require a reduction in dose to prevent toxicity. Patients should monitor their blood glucose more closely. There is no increase in allergic reactions to TMP/SMZ in patients who are diabetic. Patients taking TMP/SMZ may need reduced doses of glipizide but should not stop taking the drug.

A patient with bronchitis is taking TMP/SMZ, 160/800 mg orally, twice daily. Before administering the third dose, the nurse notes that the patient has a widespread rash, a temperature of 103°F, and a heart rate of 100 beats per minute. The patient looks ill and reports not feeling well. What will the nurse do? a. Administer the dose and request an order for an antipyretic medication. b. Withhold the dose and request an order for an antihistamine to treat the rash. c. Withhold the dose and notify the provider of the symptoms. d. Request an order for intravenous TMP/SMZ, because the patient is getting worse.

ANS: C The most severe hypersensitivity reaction with TMP/SMZ is Stevens-Johnson syndrome, which manifests with fever, malaise, and rash. The drug should be discontinued immediately if a rash occurs. Requesting an antipyretic while giving the drug is incorrect. Giving antihistamines is not indicated. Giving TMP/SMZ intravenously would make the reaction worse.

A nurse is caring for a patient who takes an ACE inhibitor and an ARB medication who will begin taking TMP/SMZ to treat a urinary tract infection. Which serum electrolyte will the nurse expect to monitor closely? a. Calcium b. Chloride c. Potassium d. Sodium

ANS: C Trimethoprim suppresses renal excretion of potassium, increasing the risk of hyperkalemia. Patients at greatest risk are those taking high doses of trimethoprim and those taking other drugs that elevate potassium, including ACE inhibitors and ARB medications. Trimethoprim does not affect other serum electrolytes.

An immunocompromised child is exposed to chickenpox and the provider orders valacyclovir [Valtrex] to be given orally three times daily. The nurse will contact the provider to change this order for which reason? a.Valacyclovir is not used as varicella prophylaxis. b.The dosage is too high for this indication. c.The drug may cause serious adverse effects in immunocompromised patients. d.Valacyclovir is not approved for use in children.

ANS: C Valacyclovir is approved for use for varicella in immunocompetent children. In immunocompromised patients, it has produced a syndrome known as thrombotic thrombocytopenic purpura/hemolytic uremia syndrome (TTP/HUS). The dosage is fine for immunocompetent children.

7. A patient who is taking doxycycline for a serious infection contacts the nurse to report anal itching. The nurse will contact the provider to discuss: a. adding an antihistamine to the patient's drug regimen. b. ordering liver function tests to test for hepatotoxicity. c. prescribing an antifungal drug to treat a superinfection. d. testing the patient for a C. difficile secondary infection.

ANS: C A superinfection occurs secondary to suppression of drug-sensitive organisms. Overgrowth with fungi, especially Candida albicans, is common and may occur in the mouth, pharynx, vagina, and bowel. Anal itching is a sign of such an infection, not a sign of hepatotoxicity. Antihistamines will not treat the cause. C. difficile is characterized by profuse, watery diarrhea.

6. A patient is to begin taking doxycycline to treat a rickettsial infection. Which statement by the patient indicates a need for teaching about this drug? a. "I should consult my provider before using laxatives or antacids while taking this drug." b. "I should not take a calcium supplement or consume dairy products with this drug." c. "I should take this drug with food to ensure more complete absorption." d. "If I get diarrhea, I should stop taking the drug and let my provider know immediately."

ANS: C Absorption of tetracyclines is reduced in the presence of food. The tetracyclines form insoluble chelates with calcium, iron, magnesium, aluminum, and zinc, so patients should not take tetracyclines with dairy products, calcium supplements, or drugs containing these minerals. Patients who experience diarrhea should stop taking the drug and notify the provider so they can be tested for C. difficile infection.

7. A patient is about to receive penicillin G for an infection that is highly sensitive to this drug. While obtaining the patient's medication history, the nurse learns that the patient experienced a rash when given amoxicillin [Amoxil] as a child 20 years earlier. What will the nurse do? a. Ask the provider to order a cephalosporin. b. Reassure the patient that allergic responses diminish over time. c. Request an order for a skin test to assess the current risk. d. Suggest using a desensitization schedule to administer the drug.

ANS: C Allergy to penicillin can decrease over time; therefore, in patients with a previous allergic reaction who need to take penicillin, skin tests can be performed to assess the current risk. Until this risk is known, changing to a cephalosporin is not necessary. Reassuring the patient that allergic responses will diminish is not correct, because this is not always the case; the occurrence of a reaction must be confirmed with skin tests. Desensitizing schedules are used when patients are known to be allergic and the drug is required

6. A patient is receiving intravenous potassium penicillin G, 2 million units to be administered over 1 hour. At 1900, the nurse notes that the dose hung at 1830 has infused completely. What will the nurse do? a. Assess the skin at the infusion site for signs of tissue necrosis. b. Observe the patient closely for confusion and other neurotoxic effects. c. Request an order for serum electrolytes and cardiac monitoring. d. Watch the patient's actions and report any bizarre behaviors.

ANS: C Although penicillin G is the least toxic of all antibiotics, certain adverse effects may be caused by compounds coadministered with penicillin. When large doses of potassium penicillin G are administered rapidly, hyperkalemia can occur, which can cause fatal dysrhythmias. When penicillin G is administered IM, tissue necrosis occurs with inadvertent intra-arterial injection. Confusion, seizures, and hallucinations can occur if blood levels of the drug are too high. Bizarre behaviors result with large IV doses of procaine penicillin G.

A nurse is reviewing the culture results of a patient receiving an aminoglycoside. The report reveals an anaerobic organism as the cause of infection. What will the nurse do? a. Contact the provider to discuss an increased risk of aminoglycoside toxicity. b. Continue giving the aminoglycoside as ordered. c. Request an order for a different class of antibiotic. d. Suggest adding a penicillin to the patient's drug regimen.

ANS: C Aminoglycosides are not effective against anaerobic microbes, so another class of antibiotics is indicated. There is no associated increase in aminoglycoside toxicity with anaerobic infection. The aminoglycoside will not be effective, so continuing to administer this drug is not indicated. Adding another antibiotic is not useful, because the aminoglycoside is not necessary.

13. A patient received 500 mg of azithromycin [Zithromax] at 0800 as a first dose. What are the usual amount and time of the second dose of azithromycin? a. 250 mg at 2000 the same day b. 500 mg at 2000 the same day c. 250 mg at 0800 the next day d. 500 mg at 0800 the next day

ANS: C Azithromycin generally is given as 500 mg on the first day and then 250 mg/day for the next 4 days, so the second dose would be 24 hours after the first dose.

3. A patient recently began receiving clindamycin [Cleocin] to treat an infection. After 8 days of treatment, the patient reports having 10 to 15 watery stools per day. What will the nurse tell this patient? a. The provider may increase the clindamycin dose to treat this infection. b. This is a known side effect of clindamycin, and the patient should consume extra fluids. c. The patient should stop taking the clindamycin now and contact the provider immediately. d. The patient should try taking Lomotil or a bulk laxative to minimize the diarrheal symptoms.

ANS: C Clostridium difficile-associated diarrhea (CDAD) is the most severe toxicity of clindamycin; if severe diarrhea occurs, the patient should be told to stop taking clindamycin immediately and to contact the provider so that treatment with vancomycin or metronidazole can be initiated. Increasing the dose of clindamycin will not treat this infection. Consuming extra fluids while still taking the clindamycin is not correct, because CDAD can be fatal if not treated. Taking Lomotil or bulk laxatives only slows the transit of the stools and does not treat the cause.

9. A 6-week-old infant who has not yet received immunizations develops a severe cough. While awaiting nasopharyngeal culture results, the nurse will expect to administer which antibiotic? a. Clindamycin [Cleocin] b. Doxycycline [Vibramycin] c. Erythromycin ethylsuccinate d. Penicillin G

ANS: C Erythromycin is the drug of first choice for infections caused by Bordetella pertussis, the causative agent of whooping cough. Infants who have not received their first set of immunizations are at increased risk of pertussis. Clindamycin, doxycycline, and penicillin are not recommended.

16. A patient who has been taking linezolid [Zyvox] for 6 months develops vision problems. What will the nurse do? a. Reassure the patient that this is a harmless side effect of this drug. b. Tell the patient that blindness is likely to occur with this drug. c. Tell the patient that this symptom is reversible when the drug is discontinued. d. Tell the patient to take tyramine supplements to minimize this effect.

ANS: C Linezolid is associated with neuropathy, including optic neuropathy. This is a reversible effect that will stop when the drug is withdrawn. Reassuring the patient that this is a harmless side effect is not correct. It is not an indication that blindness will occur. Tyramine supplements are not indicated.

A patient shows signs and symptoms of conjunctivitis. Which aminoglycoside would the nurse expect to be ordered? a. Amikacin [Amikin] b. Kanamycin [Kantrex] c. Neomycin [Neomycin] d. Paromomycin [Humatin]

ANS: C Neomycin is used for topical treatment of infections of the eye, ear, and skin. Amikacin, kanamycin, and paromomycin are not topical treatments and are not indicated for eye infections.

11. A patient with an infection caused by Pseudomonas aeruginosa is being treated with piperacillin. The nurse providing care reviews the patient's laboratory reports and notes that the patient's blood urea nitrogen and serum creatinine levels are elevated. The nurse will contact the provider to discuss: a. adding an aminoglycoside. b. changing to penicillin G. c. reducing the dose of piperacillin. d. ordering nafcillin.

ANS: C Patients with renal impairment should receive lower doses of piperacillin than patients with normal renal function. Aminoglycosides are nephrotoxic. Penicillin G and nafcillin are not effective against Pseudomonas infections.

9. A patient has an infection caused by Streptococcus pyogenes. The prescriber has ordered dicloxacillin PO. What will the nurse do? a. Administer the medication as ordered. b. Contact the provider to suggest giving the drug IV. c. Question the need for a penicillinase-resistant penicillin. d. Suggest ordering vancomycin to treat this infection.

ANS: C Penicillinase-resistant penicillins have been developed for use against penicillinase-producing strains of staphylococci. These drugs have a very narrow antimicrobial spectrum and should be used only for such infections. S. pyogenes can be treated with penicillin G. The nurse should question the order. It is incorrect to contact the provider to ask for IV dosing. This infection can be treated with penicillin G and not with vancomycin.

5. To prevent yellow or brown discoloration of teeth in children, tetracyclines should not be given: a. to children once the permanent teeth have developed. b. to patients taking calcium supplements. c. to pregnant patients after the fourth month of gestation. d. with dairy products or antacids.

ANS: C Tetracyclines bind to calcium in developing teeth, resulting in yellow or brown discoloration. They should not be given to pregnant women after the fourth month of gestation, because they will cause staining of deciduous teeth in the fetus. In children, discoloration occurs when tetracyclines are given between the ages of 4 and 8 years, because this is when permanent teeth are developing. Tetracycline binds with calcium, so absorption is diminished when the drug is given with calcium supplements, dairy products, or calcium-containing antacids; however, this does not affect tooth development.

13. A nurse is preparing to administer intramuscular penicillin to a patient who is infected with T. pallidum and notes that the order is for sodium penicillin G. Which action is correct? a. Administer the drug as prescribed. b. Contact the provider to discuss administering the drug intravenously. c. Contact the provider to discuss changing the drug to benzathine penicillin G. d. Request an order for piperacillin instead of penicillin G.

ANS: C The procaine and benzathine penicillin salts are absorbed slowly and are considered repository preparations. When benzathine penicillin G is injected IM, penicillin G is absorbed for weeks and is useful only against highly sensitive organisms such as T. pallidum. Sodium penicillin G is absorbed rapidly, with peak effects in 15 minutes. Administering the drug IV will not yield repository effects. Piperacillin is not used for T. pallidum IC.

1. A patient who is receiving a final dose of intravenous (IV) cephalosporin begins to complain of pain and irritation at the infusion site. The nurse observes signs of redness at the IV insertion site and along the vein. What is the nurse's priority action? a. Apply warm packs to the arm, and infuse the medication at a slower rate. b. Continue the infusion while elevating the arm. c. Select an alternate intravenous site and administer the infusion more slowly. d. Request central venous access.

ANS: C These signs indicate thrombophlebitis. The nurse should select an alternative IV site and administer the infusion more slowly. The IV should not be continued in the same site, because necrosis may occur. A central line would be indicated only for long-term administration of antibiotics.

A newborn infant has been given erythromycin ophthalmic ointment as a routine postpartummedication. The infant's mother learns that she has a C. trachomatis infection and asks the nurse if her baby will need to be treated. Which response by the nurse is correct? a. "The erythromycin ointment will prevent your baby from developing conjunctivitis." b."Without additional treatment, your baby could develop blindness." c."Your baby will need to be treated with oral erythromycin." d."Your baby will need to take doxycycline (Vibramycin) for 10 days."

ANS: C Infants born to women with cervical C. trachomatis are at risk for conjunctivitis and pneumonia. Topical erythromycin may help prevent conjunctivitis, but it is not completely effective and does not prevent pneumonia; therefore, this infant will need treatment with oral erythromycin. Blindness will not occur as a result of C. trachomatis conjunctivitis. Doxycycline is contraindicated, because it causes staining of the teeth in children under 8 years of age.

A patient is diagnosed with pelvic inflammatory disease (PID). Which treatment regimen is most appropriate for reducing the risk of sterility in this patient? a. Azithromycin (Zithromax), 1 gm PO once, and cefoxitin, 2 gm IM once in the clinic b. Ceftriaxone (Rocephin), 250 mg IM once, with doxycycline (Vibramycin), 100 mg PO twice daily for 14 days as an outpatient. c. Doxycycline (Vibramycin), 100 mg IV twice daily, and cefoxitin, 2 gm IV every 6 hours in the hospital d. Doxycycline (Vibramycin), 100 mg PO twice daily for 14 days, and metronidazole (Flagyl), 500 mg PO twice daily for 14 days in the hospital

ANS: C Many experts recommend that all patients with PID receive IV antibiotics in the hospital to minimize the risk of sterility and other complications. Medications used intravenously can be cefoxitin or cefotetan combined with doxycycline; when symptoms resolve, IV therapy may be discontinued but must be followed by PO doxycycline. Outpatient regimens are not recommended. Oral medications are not recommended for initial treatment.

A patient is admitted to the hospital with fever, headache, malaise, joint pain, and enlarged lymph nodes. Blood cultures are positive for Treponema pallidum. The nurse recognizes this as which type of syphilis? a. Congenital b. Primary c. Secondary d. Tertiary

ANS: C Secondary syphilis occurs when the organism spreads to the bloodstream, causing systemic symptoms such as fever, headache, reduced appetite, and general malaise, along with enlarged lymph nodes and joint pain. Congenital syphilis occurs when infants are exposed to T. pallidumin utero; early symptoms include sores, rhinitis, and point tenderness over bones. Primary syphilis is characterized by a primary lesion, called a chancre, at the point of entry, along with enlarged lymph nodes. Tertiary syphilis develops 5 to 40 years after the initial infection and can involve the brain, heart, and other sites.

A patient who is taking didanosine [Videx] reports nausea, vomiting, and abdominal pain. What will the nurse recommend to this patient? a."Take the drug with food to minimize these side effects." b."Stop taking the drug immediately and resume taking it once your symptoms subside." c."Take the medication in the evening to avoid experiencing these kinds of symptoms." d."You will need laboratory tests to determine if these are serious effects of the drug."

ANS: D As with all NRTIs, pancreatitis may occur and may manifest as nausea, vomiting, and abdominal pain. The patient will need evaluation of serum amylase, triglycerides, and calcium. Taking the drug with food or at a different time of day are not indicated. It is not correct to discontinue the drug and to resume it when symptoms subside, since pancreatitis may be fatal.

A pregnant patient is treated with trimethoprim/sulfamethoxazole (TMP/SMZ) [Bactrim] for a urinary tract infection at 32 weeks' gestation. A week later, the woman delivers her infant prematurely. The nurse will expect to monitor the infant for: a. birth defects. b. hypoglycemia. c. rash. d. kernicterus.

ANS: D Kernicterus is a disorder in newborns caused by deposition of bilirubin in the brain. Sulfonamides promote kernicterus by displacing bilirubin from plasma proteins. Sulfonamides should not be given to infants under 2 months of age or to pregnant women after 32 weeks' gestation. Sulfonamides do not cause birth defects or hypoglycemia. Serious rashes may occur but are not the primary concern in the newborn.

A nurse teaches a patient about sulfonamides. Which statement by the patient indicates a need for further teaching? a. "I need to drink extra fluids while taking this medication." b. "I need to use sunscreen when taking this drug." c. "I should call my provider if I develop a rash while taking this drug." d. "I should stop taking this drug when my symptoms are gone."

ANS: D Patients should always be advised to complete the prescribed course of the antibiotic even when symptoms subside. Patients should also understand the need to drink 8 to 10 glasses of water a day, to use sunscreen, and to notify the provider if they develop a rash.

After starting an antiviral protease inhibitor, a patient with HIV telephones the nurse, complaining, "I'm so hungry and thirsty all the time! I'm urinating 10 or 12 times a day." The nurse recognizes these findings to be consistent with: a.pancreatic infiltration by HIV. b.allergic reaction. c.nonadherence to the antiviral regimen. d.hyperglycemia.

ANS: D Protease inhibitors have been associated with hyperglycemia, new-onset diabetes, abrupt exacerbation of existing diabetes, and diabetic ketoacidosis. These symptoms are not consistent with pancreatic infiltration or an allergic reaction. No evidence indicates that the patient is noncompliant.

A female patient who has hepatitis C is being treated with pegylated interferon alfa and ribavirin [Ribasphere]. It will be important for the nurse to teach this patient that: a.if she gets pregnant, she should use the inhaled form of ribavirin [Virazole]. b.if she is taking oral contraceptives, she should also take a protease inhibitor. c.she should use a hormonal contraceptive to avoid pregnancy. d.she will need a monthly pregnancy test during her treatment.

ANS: D Ribavirin causes severe fetal injury and is contraindicated during pregnancy. Women taking ribavirin must rule out pregnancy before starting the drug, monthly during treatment, and monthly for 6 months after stopping treatment. Inhaled ribavirin is also embryo lethal and teratogenic. Adding a protease inhibitor will reduce the efficacy of oral contraceptives. Women using ribavirin should use two reliable forms of birth control.

A nurse is caring for an African American patient who has been admitted to the unit for long-term antibiotic therapy with sulfonamides. The patient develops fever, pallor, and jaundice. The nurse would be correct to suspect that the patient has developed: a. Stevens-Johnson syndrome. b. kernicterus. c. hepatotoxicity. d. hemolytic anemia.

ANS: D Sulfonamides can cause hemolytic anemia in patients of African American and Mediterranean origin, usually because of a genetic deficiency. Red cell lysis can produce fever, pallor, and jaundice, and patients should be observed for these signs. The patient's signs and symptoms are not characteristic of Stevens-Johnson syndrome or hepatotoxicity. The patient's signs and symptoms are not characteristic of kernicterus, which occurs in newborns.

A nurse is obtaining a drug history from a patient about to receive sulfadiazine. The nurse learns that the patient takes warfarin, glipizide, and a thiazide diuretic. Based on this assessment, the nurse will expect the provider to: a. change the antibiotic to TMP/SMZ. b. increase the dose of the glipizide. c. monitor the patient's electrolytes closely. d. monitor the patient's coagulation levels.

ANS: D Sulfonamides interact with several drugs and through metabolism-related interactions can intensify the effects of warfarin. Patients taking both should be monitored closely for bleeding tendencies. Changing to the combination product will not help, because sulfonamides are still present. Sulfonamides intensify glipizide levels, so this drug may actually need to be reduced. Trimethoprim, not sulfonamides, raises potassium levels.

A patient starting therapy with efavirenz [Sustiva] asks about the timing of the medication with regard to meals. What patient education about the administration of this medication should the nurse provide? a.The drug must be taken within 30 minutes after a meal. b.The drug is best taken with a high-fat meal. c.The drug can be taken anytime without regard to meals. d.The drug should be taken once daily on an empty stomach.

ANS: D The nurse should advise the patient that the medication should be taken once daily on an empty stomach. Thirty minutes after a meal is too soon to take the medication. The medication is taken on an empty stomach, because high-fat meals increase plasma levels by 39% with capsules and by 79% with tablets. The medication must not be taken with high-fat meals.

The nurse is caring for a patient who is HIV positive and is taking zidovudine [Retrovir]. Before administering the medication, the nurse should monitor which laboratory values? a.Ketones in the urine and blood b.Serum immunoglobulin levels c.Serum lactate dehydrogenase d.Complete blood count (CBC)

ANS: D The nurse should monitor the patient's CBC to determine whether the patient has anemia and neutropenia. Ketones are not an adverse effect of zidovudine. Nothing indicates a need to monitor the immunoglobulin levels or serum lactate dehydrogenase.

A nurse is discussing microbial resistance among sulfonamides and trimethoprim with a nursing student. Which statement by the student indicates a need for further teaching? a. "Bacterial resistance to trimethoprim is relatively uncommon." b. "Resistance among gonococci, streptococci, and meningococci to sulfonamides is high." c. "Resistance to both agents can occur by spontaneous mutation of organisms." d. "Resistance to sulfonamides is less than resistance to trimethoprim."

ANS: D There is less microbial resistance to trimethoprim than there is to sulfonamides. Bacterial resistance to trimethoprim is relatively uncommon. Gonococcal, streptococcal, and meningococcal resistance to sulfonamides is especially high. For both agents, resistance can develop by spontaneous mutation.

10. The parent of an infant with otitis media asks the nurse why the prescriber has ordered amoxicillin [Amoxil] and not ampicillin [Unasyn]. What will the nurse tell the parent? a. Amoxicillin is a broader spectrum antibiotic than ampicillin. b. Amoxicillin is not inactivated by beta-lactamases. c. Ampicillin is associated with more allergic reactions. d. Ampicillin is not as acid stable as amoxicillin.

ANS: D Amoxicillin and ampicillin are similar in structure and actions but differ primarily in acid stability. Amoxicillin is more acid stable and, when administered orally, results in higher blood levels than can be obtained with equivalent doses of ampicillin. The two drugs have the same spectrum, both are inactivated by beta-lactamases, and both can cause allergic reactions.

4. A child with otitis media has had three ear infections in the past year. The child has just completed a 10-day course of amoxicillin [Amoxil] with no improvement. The parent asks the nurse why this drug is not working, because it has worked in the past. What will the nurse tell the patient? a. "Amoxicillin is too narrow in spectrum." b. "The bacteria have developed a three-layer cell envelope." c. "The bacteria have developed penicillin-binding proteins (PBPs) that have a low affinity for penicillins." d. "The bacteria have synthesized penicillinase."

ANS: D Beta-lactamases are enzymes that cleave the beta-lactam ring and render the PCN inactive. This resistance is common with organisms that cause ear infections. Amoxicillin is a broad-spectrum antibiotic. A three-layer cell envelope occurs in gram-negative bacteria. Some bacterial strains, including methicillin-resistant Staphylococcus aureus (MRSA), develop PBPs with a low affinity for penicillins. MRSA is not a common cause of otitis media.

4. A patient who is taking calcium supplements receives a prescription for ciprofloxacin [Cipro] for a urinary tract infection. The nurse will teach this patient to: a. consume extra fluids while taking the ciprofloxacin to prevent hypercalciuria. b. stop taking the calcium supplements while taking the ciprofloxacin. c. take the two medications together to increase the absorption of both. d. take the calcium either 6 hours before or 2 hours after taking the ciprofloxacin.

ANS: D Cationic compounds, including calcium supplements, can reduce the absorption of ciprofloxacin, so proper interval dosing is necessary. Consuming extra fluids is not indicated. With proper interval dosing, it is not necessary to discontinue the calcium while giving the ciprofloxacin. These two medications should not be given together.

2. Which cephalosporin may be used to treat meningitis? a. Cefaclor b. Cefazolin c. Cefoxitin d. Cefotaxime

ANS: D Cefotaxime has increased ability to reach the cerebrospinal fluid (CSF) and to treat meningitis. Cefaclor, cefazolin, and cefoxitin do not reach effective concentrations in the CSF.

11. A patient has a skin infection and the culture reveals methicillin-resistant Staphylococcus aureus (MRSA). What is an appropriate treatment for this patient? a. Cefaclor b. Cefazolin c. Cefotaxime d. Ceftaroline

ANS: D Ceftaroline is a fifth-generation cephalosporin with a spectrum similar to third-generation cephalosporins but also with activity against MRSA. Cefaclor is a second-generation cephalosporin. Cefazolin is a first-generation cephalosporin. Cefotaxime is a third-generation cephalosporin.

8. The nurse is caring for a patient who will begin receiving intravenous ciprofloxacin [Cipro] to treat pyelonephritis. The nurse learns that the patient has a history of myasthenia gravis. Which action by the nurse is correct? a. Administer the ciprofloxacin and monitor the patient for signs of muscle weakness. b. Ask the provider whether the ciprofloxacin can be given orally. c. Request an order for concurrent administration of metronidazole [Flagyl]. d. Suggest that the provider order a different antibiotic for this patient.

ANS: D Ciprofloxacin and other fluoroquinolones can exacerbate muscle weakness in patients with myasthenia gravis and should not be given to these patients. It is not correct to administer the drug and monitor for this effect. Giving the drug by a different route will not alter this effect. Metronidazole is given when C. difficile occurs.

8. A hospitalized patient who is taking demeclocycline [Declomycin] reports increased urination, fatigue, and thirst. What will the nurse do? a. Contact the provider to report potential toxic side effects. b. Notify the provider to discuss changing the medication to doxycycline. c. Perform bedside glucometer testing to evaluate the serum glucose level. d. Provide extra fluids and reassure the patient that these are expected side effects.

ANS: D Demeclocycline stimulates urine flow and sometimes is used to treat patients with syndrome of inappropriate antidiuretic hormone secretion (SIADH). The patient should be reassured that increased urination, fatigue, and thirst are known side effects of demeclocycline. It is not correct to notify the provider of toxic side effects or to request another tetracycline. Glucometer testing is not necessary, because the increased urination is not related to an elevated blood glucose level.

18. A child has been receiving chloramphenicol for a Neisseria meningitidis central nervous system (CNS) infection. The nurse administers the dose and subsequently notes that the child has vomited and appears dusky and gray in color. The child's abdomen is distended. What will the nurse do? a. Contact the provider for an order to obtain a chloramphenicol level. b. Notify the provider that the child's meningitis is worsening. c. Recognize this as initial signs of a C. difficile infection. d. Stop the infusion immediately and notify the provider.

ANS: D Gray syndrome is a potentially fatal toxicity associated with chloramphenicol use. When symptoms occur, the drug should be stopped immediately. Lower chloramphenicol levels may prevent gray syndrome, but lowering the dose will not stop symptoms once they have appeared. These are not signs of worsening meningitis or a C. difficile infection.

7. A provider orders intravenous moxifloxacin [Avelox] for a patient who has sinusitis. Before administering the drug, the nurse will review this patient's chart for: a. a history of asthma. b. concurrent use of digoxin. c. concurrent use of warfarin. d. recent serum electrolyte levels.

ANS: D Moxifloxacin prolongs the QT interval and poses a risk of serious dysrhythmias. Patients with hypokalemia have an increased risk, so serum electrolyte levels should be monitored. Having a history of asthma is not significant. Moxifloxacin does not alter digoxin or warfarin levels.

5. A nurse is preparing to administer oral ofloxacin to a patient. While taking the patient's medication history, the nurse learns that the patient takes warfarin and theophylline. The correct action by the nurse is to request an order to: a. reduce the dose of ofloxacin. b. increase the dose of ofloxacin. c. increase the dose of theophylline. d. monitor coagulation levels.

ANS: D Ofloxacin increases plasma levels of warfarin, so coagulation tests should be monitored. The ofloxacin dose should not be reduced or increased. Ofloxacin does not affect theophylline levels.

A nurse preparing to administer intravenous gentamicin to a patient notes that the dose is half the usual dose for an adult. The nurse suspects that this is because this patient has a history of: a. antibiotic resistance. b. interpatient variation. c. liver disease. d. renal disease.

ANS: D The aminoglycosides are eliminated primarily by the kidneys, so in patients with renal disease, doses should be reduced or the dosing interval should be increased to prevent toxicity. Patients with antibiotic resistance would be given amikacin. Interpatient variation may occur but cannot be known without knowing current drug levels. Aminoglycosides are not metabolized by the liver, so liver disease would not affect drug levels.

12. A patient who has cystic fibrosis has a Pseudomonas aeruginosa infection and the provider has ordered aztreonam [Cayston]. What will the nurse teach this patient about administration of this drug? a. Administer the drug intramuscularly twice daily. b. Give a daily dose every day for 28 days and then stop. c. Inhale the powdered drug as ordered three times each day. d. Use the nebulizer to administer the drug three times daily.

ANS: D Cayston is a form of aztreonam formulated for inhalation administration for patients with cystic fibrosis who have P. aeruginosa lung infections. The reconstituted powder is given using a nebulizer system three times daily for 28 days followed by 28 days off. This form of the drug is not given IM. The dose is three times daily. The drug is reconstituted and administered via a nebulizer.

A nurse is teaching a group of adolescent students about sexually transmitted diseases. Which statement by a student indicates understanding of infections caused by C. trachomatis? a. "Chlamydia trachomatis conjunctivitis in newborns can result in blindness." b."The CDC recommends screening for chlamydial infections in all sexually active men." c."Treatment for C. trachomatis should be initiated when infections are symptomatic." d."Women with asymptomatic Chlamydia trachomatis infections can become

ANS: D Chlamydial infections are frequently asymptomatic in women and may be asymptomatic in men; they can cause sterility in women. C. trachomatis conjunctivitis in newborns does not result in blindness. The Centers for Disease Control and Prevention (CDC) recommends routine screening in sexually active women under age 25 and for those over age 25 with new partners or multiple partners. Treatment should begin when infections are identified, regardless of whether they are symptomatic.

A patient has a viral sinus infection, and the provider tells the patient that antibiotics will not be prescribed. The patient wants to take an antibiotic and asks the nurse what possible harm could occur by taking an antibiotic. Which response by the nurse is correct? A) "Antibiotics are mutagenic and can produce changes that cause resistance," B) "Even normal flora can develop resistance and transfer this to pathogens." C) "Host cells become resistant to antibiotics with repeated use." D) "Patients who overuse antibiotics are more likely to have nosocomial infections."

B Antibiotics make conditions favorable for the overgrowth of microbes with acquired resistance. Normal flora, present at all times, can develop resistance and can transfer this resistance to pathogens if they occur. Even when pathogens are not present, antibiotic use can promote resistance in the future. Antibiotics are not mutagenic. Host cells are not affected. Antibiotic use does not increase the risk of nosocomial infection in a particular patient but does increase resistance in resident organisms in a particular hospital.

A child has received amoxicillin (Amoxil) for three previous ear infections, but a current otitis media episode is not responding to treatment. The nurse caring for this child suspects that resistance to the bacterial agent has occurred by which microbial mechanism? a. alteration of drug target molecules b. antagonist production c. drug inactivation d. reduction of drug concentration at the site of action

B Drug inactivation can occur when microbes produce drug-metabolizing enzymes. Penicillin- resistant organisms, including many that cause otitis media, produce penicillinase. Alteration of drug target molecules, drug inactivation, and reduction of the drug concentration occur with other antimicrobials.

A patient with a history of congestive heart failure and renal impairment has esophageal candidiasis. Which antifungal agent would the nurse anticipate giving to this patient? a. Amphotericin B [Abelcet] b. Fluconazole [Diflucan] c. Itraconazole [Sporanox] d. Voriconazole [Vfend]

B Fluconazole is a drug of choice for treating systemic candidal infections. Amphotericin is nephrotoxic and should not be used in patients with existing renal disease. Itraconazole is a possible alternative agent for treating candidiasis but has serious cardiac side effects. Voriconazole is a drug of first choice for treating aspergillosis but not for candidiasis.

A patient who is taking ketoconazole tells the nurse that her periods have become irregular. What will the nurse tell her? a. This indicates that she should begin taking oral contraceptives. b. This is caused by a reversible effect on estradiol synthesis. c. This is a serious side effect that warrants discontinuation of the drug. d. This is a sign of hepatic toxicity, and the drug dose should be lowered.

B Ketoconazole inhibits steroid synthesis in humans, and in females it reduces estradiol synthesis, causing menstrual irregularities. It is not an indication that oral contraceptive pills (OCPs) are needed. Because it is reversible, there is no indication for withdrawing the drug until treatment is complete. Menstrual irregularities are not a sign associated with hepatotoxicity.

A nurse is caring for a patient who is receiving amphotericin B [Abelcet] for a systemic fungal infection. In spite of receiving diphenhydramine and acetaminophen before initiation of treatment, the patient has fever and chills with rigors. The nurse will contact the provider to discuss the addition of which drug? a. Aspirin b. Dantrolene c. Hydrocortisone d. Omeprazole

B Patients receiving amphotericin frequently experience infusion reactions, with fever, chills, rigors, nausea, and headache. Pretreatment with diphenhydramine and acetaminophen can reduce mild reactions. If rigors occur, the patient should receive dantrolene or meperidine. Aspirin would be effective for pretreatment but can increase kidney damage. Hydrocortisone is also effective, but it causes immune suppression. Omeprazole is not indicated.

An older adult patient with chronic obstructive pulmonary disease (COPD) develops bronchitis. The patient has a temperature of 39.5° C. The nurse will expect the provider to: A) obtain a sputum culture and wait for the results before prescribing an antibiotic. B) order empiric antibiotics while waiting for sputum culture results. C) treat symptomatically, because antibiotics are usually ineffective against bronchitis. D) treat the patient with more than one antibiotic without obtaining cultures.

B Patients with severe infections should be treated while culture results are pending. If a patient has a severe infection or is at risk of serious sequelae if treatment is not begun immediately, it is not correct to wait for culture results before beginning treatment. Until a bacterial infection is ruled out, treating symptomatically is not indicated. Treating without obtaining cultures is not recommended.

A patient is receiving intravenous voriconazole [Vfend]. Shortly after the infusion starts, the patient tells the nurse, "Colors look different, and the light hurts my eyes." What will the nurse do? a. Observe the patient closely for the development of hallucinations. b. Reassure the patient that these effects will subside in about 30 minutes. c. Stop the infusion and notify the provider of CNS toxicity. d. Tell the patient that this is an irreversible effect of the drug.

B Reduced visual acuity, increased brightness, altered color perception, and photophobia are reversible visual disturbances that can occur in 30% of patients receiving voriconazole. These usually begin within 30 minutes of dosing and then diminish over the next 30 minutes. They are not precursors to other effects, such as hallucinations. They do not indicate CNS toxicity. They are reversible.

A patient is taking oral ketoconazole [Nizoral] for a systemic fungal infection. The nurse reviews the medication administration record and notes that the patient is also taking omeprazole [Prilosec] for reflux disease. What action should the nurse take? a. Administer the omeprazole 1 hour before the ketoconazole. b. Administer the omeprazole at least 2 hours after the ketoconazole. c. Confer with the prescriber about a potential hazardous interaction. d. The nurse should not administer omeprazole to a patient receiving ketoconazole.

B The nurse should administer the omeprazole at least 2 hours after the ketoconazole to prevent a drug-to-drug interaction. Drugs that reduce gastric acidity should be administered no sooner than 2 hours after ingestion of ketoconazole, because they reduce absorption of the drug. There is no need to confer with the prescriber or to refuse to administer the drugs, because ketoconazole and omeprazole do not have a potentially hazardous interaction.

A patient with histoplasmosis is being treated with itraconazole [Sporanox]. The nurse will teach this patient to report which symptoms? a. Gynecomastia and decreased libido b. Headache and rash c. Nausea, vomiting, and anorexia d. Visual disturbances

C Itraconazole is associated with rare cases of liver failure, some of which were fatal. Patients should be instructed to report signs of liver toxicity, including nausea, vomiting, and anorexia. Ketoconazole is associated with gynecomastia and libido changes. Headache and rash are associated with fluconazole. Visual disturbances may occur with voriconazole

A patient will begin taking an immunosuppressant medication. The nurse learns that the patient has a history of frequent candidal infections. The nurse will expect the provider to order which drug as prophylaxis? a. Fluconazole [Diflucan] b. Ketoconazole c. Posaconazole [Noxafil] d. Voriconazole [Vfend]

C Posaconazole is used as prophylaxis for invasive Aspergillus and Candida infections in immunocompromised patients. Fluconazole, ketoconazole, and voriconazole are not used prophylactically.

A nursing student asks a nurse to clarify the differences between the mechanisms of spontaneous mutation and conjugation in acquired resistance of microbes. What will the nurse say? a. conjugation result in a gradual increase in resistance b. conjugation results in random changes in the microbe's DNA c. spontaneous mutation leads to resistance to only one antimicrobial agent d. spontaneous mutation can transfer DNA from one organism to another

C Spontaneous mutation generally confers resistance to only one drug. Conjugation can occur quickly; spontaneous mutation is gradual. Spontaneous mutation is random; conjugation is not. Conjugation can occur with the transfer of DNA from one organism to another

A recent campaign, initiated by the Centers for Disease Control (CDC), to delay the emergence of antibiotic resistance in hospitals, has what as one of its objectives? a. allowing patient to stop antibiotics when symptoms subside b. allowing prescribers to develop their own prescribing guidelines c. increased adherence to prescribed antibiotics d. increased use of antibiotics among parents of young children

C The CDC initiative has identified three objectives to help delay the emergence of resistance to antibiotics in hospitals. One of the objectives is to increase adherence to prescribed antibiotics among users. Allowing patients to stop using antibiotics before the prescription ends is not one of the objectives and may increase the risk of resistance. Other objectives include ensuring that prescribers adhere to appropriate prescribing guidelines instead of developing their own and decreasing antibiotic use among parents of small children.D

A nursing student asks a nurse to explain the differences between amphotericin B [Abelcet] and the azoles group of antifungal agents. Which statement by the nurse is correct? a. "Amphotericin B can be given orally or intravenously." b. "Amphotericin B increases the levels of many other drugs." c. "Azoles have lower toxicity than amphotericin B." d. "Only the azoles are broad-spectrum antifungal agents."

C The azoles class of antifungals is less toxic than amphotericin B. Amphotericin B may only be given parenterally. The azoles, not amphotericin B, inhibit hepatic P450 drug-metabolizing enzymes, so they increase the levels of many other drugs. Both classes are broad-spectrum antifungal agents.

A nursing student asks a nurse about flucytosine [Ancobon]. Which statement by the nurse is correct? a. "Flucytosine has a broad antifungal spectrum." b. "Irreversible neutropenia and thrombocytopenia may occur with this drug." c. "Resistance is common with this medication." d. "Severe hepatic injury is common and limits this drug's use."

C The development of resistance during flucytosine therapy is common and is a serious clinical problem. Flucytosine has a narrow antifungal spectrum. Neutropenia and thrombocytopenia may occur but are reversible. Severe hepatic injury is rare; mild and reversible hepatic dysfunction is common

A child has ringworm of the scalp. A culture of the lesion reveals a dermatophytic infection. The nurse teaching the child's parents about how to treat this infection will include which statement? a. "Adverse effects of the medication include itching, burning, and erythema." b. "Apply the topical medication daily until at least 1 week after the rash is gone." c. "Your child will need to take this oral medication for 6 to 8 weeks." d. "You will use an antifungal shampoo to treat this infection."

C Tinea capitis is difficult to treat with topical medications; oral griseofulvin, taken for 6 to 8 weeks, is standard therapy. Itching, burning, and erythema are side effects of topical agents. Patients treating tinea corporis, tinea cruris, or tinea pedis are taught to apply medication until 7 days after the rash disappears. Antifungal shampoos are not effective for treating tinea capitis.

A patient will begin treatment with posaconazole [Noxafil] to treat oropharyngeal candidiasis that has not responded to fluconazole. The provider has ordered 200 mg three times daily. Which action by the nurse is correct? a. Administer the drug as ordered. b. Contact the provider to discuss decreasing the dose to twice daily. c. Discuss a 400-mg twice daily drug regimen with the provider. d. Suggest to the provider that this dose may be too high for this indication.

C When given for treatment of oropharyngeal candidiasis that is refractory to treatment with itraconazole or fluconazole, the dose is 400 mg twice daily. A dosage regimen of 200 mg three times daily is used for prophylaxis of invasive fungal infections and decreasing this to twice daily is not correct.

The nurse is teaching a nursing student about the mechanism by which antimicrobial agents achieve selective toxicity. Which statement by the student indicates a need for further teaching? A) "Some agents disrupt the bacterial cell wall;" B) "Some agents act to block the conversion of para-aminobenzoic acid (PABA) to folic acid." C) "Some agents cause phagocytosis of bacterial cells." D) "Some agents weaken the cell wall, causing cell wall lysis."

C Whenever possible, the infecting organism should be identified before antibiotics are started, even if there is a strong suspicion that a particular organism is present. The nurse is correct in telling the parent to bring the child to the clinic for a throat culture. Fever and sore throat have other causes, so it is not correct to treat this child empirically, especially because these symptoms are not severe or life-threatening. Although good hand washing is always indicated, this child already has symptoms and needs to be evaluated. Antibiotics should not be started until indicated by cultures.

The parent of a 5-year-old child who has had four urinary tract infections in the past year asks the nurse why the provider doesn't just order an antibiotic for the child's current symptoms of low-grade fever, flank pain, and dysuria since these are similar symptoms as before. Which is the most important reason given by the nurse? a. "Your child may need to be hospitalized for treatment." b. "Your child may need a urine culture before and after treatment." c. "Your child may need tests to assess for urinary tract abnormalities." d. "Your child may need additional medications, such as urinary tract antiseptics."

C Children with recurrent urinary tract infections should be assessed for underlying urinary tract abnormalities to help determine a possible cause for recurrence. This child has mild fever and therefore may not require hospitalization. Urine cultures are important when treating patients with recurrent UTI prophylactically, but this is not the most important consideration. Urinary tract antiseptics are used to treat uncomplicated lower urinary tract infections.

A pregnant female patient with bacteriuria, suprapubic pain, urinary urgency and frequency, and a low-grade fever is allergic to sulfa, ciprofloxacin, and amoxicillin. The nurse knows that the best alternative for treating this urinary tract infection is with: a. cephalexin [Keflex]. b. fosfomycin [Monurol]. c. methenamine [Hiprex]. d. nitrofurantoin [Macrodantin].

C Methenamine is an excellent second-line drug for this patient and is indicated because of the patient's multiple drug sensitivities. It is safe in pregnancy, and there is no drug resistance. Nitrofurantoin has potential harmful effects on the fetus and should not be used during pregnancy. Single-dose regimens are not recommended in pregnant women. Cephalexin can have cross-reactivity with amoxicillin.

A patient presents to the emergency department with complaints of chills, severe flank pain, dysuria, and urinary frequency. The patient has a temperature of 102.9°F, a pulse of 92 beats per minute, respirations of 24 breaths per minute, and a blood pressure of 119/58 mm Hg. The nurse would be correct to suspect that the patient shows signs and symptoms of: a. acute cystitis. b. urinary tract infection. c. pyelonephritis. d. prostatitis.

C The nurse should suspect pyelonephritis. Pyelonephritis is characterized by fever, chills, severe flank pain, dysuria, urinary urgency and frequency, and pyuria and bacteriuria. Clinical manifestations of acute cystitis include dysuria, urinary urgency and frequency, suprapubic discomfort, pyuria, and bacteriuria. Urinary tract infections (UTIs) are very general and are classified by their location. These symptoms are specific to pyelonephritis. Prostatitis is manifested by high fever, chills, malaise, myalgia, localized pain, and various UTI symptoms but not by severe flank pain.

A 30-year-old male patient reports having two to four urinary tract infections a year. What will the nurse expect to teach this patient? a. "Make sure you void after intercourse and drink extra fluids to stay well hydrated." b. "We will treat each infection as a separate infection and treat with short-course therapy." c. "You will need to take a low dose of medication for 6 months to prevent infections." d. "You will need to take antibiotics for 4 to 6 weeks each time you have an infection."

C This patient has reinfection of his urinary tract at a rate of more than three per year, which is an indication for long-term prophylaxis. Voiding after intercourse is a good teaching point for sexually active women to prevent urinary infections, but it is not a sufficient preventive measure for recurrent infections in men. Short-course therapy may be used for each occurrence of infection if the reinfection rate is less than three per year. Long-term treatment for individual infections is recommended if relapse occurs or if infections do not clear with shorter-term therapy.

A patient has come to the clinic with tinea corporis, and the prescriber has ordered clotrimazole. When educating the patient about this medication, the nurse will include which statement? a. "Apply the medication over the entire body twice daily for 2 weeks." b. "Sun exposure will minimize the drug's effects." c. "This drug is effective after a single application." d. "Use the medication for at least 1 week after the symptoms have cleared."

D The nurse should advise the patient to continue therapy for at least 1 week after the symptoms have cleared up. The medication should be applied only to the affected areas. Sun exposure will not delay the effects of clotrimazole. The drug must be applied twice daily for several weeks

A provider has ordered oral voriconazole [Vfend] for a patient who has a systemic fungal infection. The nurse obtains a medication history and learns that the patient takes phenobarbital for seizures. The nurse will contact the provider to discuss which possibility? a. Administering intravenous voriconazole b. Reducing the dose of phenobarbital c. Reducing the dose of voriconazole d. Using a different antifungal agent

D Voriconazole can interact with many drugs. It should not be combined with drugs that are powerful P450 inhibitors, including phenobarbital, because these can reduce the levels of voriconazole. Administering the voriconazole IV will not increase the serum level. It is not correct to reduce the dose of either drug.

A patient has an invasive aspergillosis infection. Which antifungal agent is the drug of choice for this infection? a. Amphotericin B b. Fluconazole [Diflucan] c. Posaconazole [Noxafil] d. Voriconazole [Vfend]

D Voriconazole has replaced amphotericin B as the drug of choice for treating invasive aspergillosis. Fluconazole, which is fungistatic, is not used to treat aspergillosis. Posaconazole is used for prophylaxis of aspergillosis in immunocompromised patients.

A patient has a localized skin infection, which is most likely caused by a gram-positive cocci. Until the culture and sensitivity results are available, the nurse will expect the provider to order a ____ spectrum _____ agent. A) broad; systemic B) broad; topical C) narrow; systemic D) narrow; topical

D When infections are treated before the causative agent has been identified, and after cultures have been obtained, antibiotics may be used based on the knowledge of which microbes are most likely to cause infection at that particular site. Because this is a localized infection, a topical agent is recommended. Unless the infection is very serious, a narrow-spectrum antibiotic is best

A parent asks a nurse if the provider will prescribe an antibiotic for a child who attends school with several children who have strep throat. The child is complaining of a sore throat and has a fever. What will be the nurse's response? A) "Because strep throat is likely, your child should be treated empirically." B) "With good hand washing, your child should not get strep throat." C) "Your child probably has strep throat, so your provider will order an antibiotic." D) "Your child should come to the clinic to have a throat culture done today."

D Whenever possible, the infecting organism should be identified before antibiotics are started, even if there is a strong suspicion that a particular organism is present. The nurse is correct in telling the parent to bring the child to the clinic for a throat culture. Fever and sore throat have other causes, so it is not correct to treat this child empirically, especially because these symptoms are not severe or life threatening. Although good hand washing is always indicated, this child already has symptoms and needs to be evaluated. Antibiotics should not be started until indicated by cultures.

A young, nonpregnant female patient with a history of a previous urinary tract infection is experiencing dysuria, urinary urgency and frequency, and suprapubic pain of 3 days' duration. She is afebrile. A urine culture is positive for more than 100,000/mL of urine. The nurse caring for this patient knows that which treatment is most effective? a. A 14-day course of amoxicillin with clavulanic acid [Augmentin] b. A 7-day course of ciprofloxacin [Cipro] c. A single dose of fosfomycin [Monurol] d. A 3-day course of trimethoprim/sulfamethoxazole [Bactrim]

D Short-course therapy is recommended for uncomplicated, community-acquired lower urinary tract infections. The short course is more effective than a single dose, and compared with longer-course therapies, it is less costly, has fewer side effects, and is more likely to foster compliance. Amoxicillin with clavulanic acid is a second-line drug used for pyelonephritis. Fosfomycin is a second-line drug and can be useful in patients with drug allergies.

An older male patient comes to the clinic with complaints of chills, malaise, myalgia, localized pain, dysuria, nocturia, and urinary retention. The nurse would most likely suspect that the patient has: a. acute cystitis. b. urinary tract infection. c. pyelonephritis. d. prostatitis.

D The nurse should suspect prostatitis, which is manifested by high fever, chills, malaise, myalgia, and localized pain and may also be manifested by dysuria, nocturia, and urinary urgency, frequency, and retention. Clinical manifestations of acute cystitis include dysuria, urinary urgency and frequency, suprapubic discomfort, pyuria, and bacteriuria. Urinary tract infections are very general and are classified by their location. Pyelonephritis is characterized by fever, chills, severe flank pain, dysuria, and urinary frequency and urgency, as well as by pyuria and bacteriuria.

Which patient with a urinary tract infection will require hospitalization and intravenous antibiotics? a. A 5-year-old child with a fever of 100.5°F, dysuria, and bacteriuria b. A pregnant woman with bacteriuria, suprapubic pain, and fever c. A young man with dysuria, flank pain, and a previous urinary tract infection d. An older adult man with a low-grade fever, flank pain, and an indwelling catheter

D The patient with an indwelling catheter and signs of pyelonephritis shows signs of a complicated UTI, which is best treated with intravenous antibiotics. The other three patients show signs of uncomplicated urinary tract infections that are not severe and can be treated with oral antibiotics.

A patient who is taking nitrofurantoin calls the nurse to report several side effects. Which side effect of this drug causes the most concern and would require discontinuation of the medication? a. Anorexia, nausea, and vomiting b. Brown-colored urine c. Drowsiness d. Tingling of the fingers

D Tingling of the fingers can indicate peripheral neuropathy, which can be an irreversible side effect of nitrofurantoin. The other side effects are not serious and can be reversed.

A patient is about to begin treatment for latent tuberculosis. The patient is an alcoholic, has difficulty complying with drug regimens, and has mild liver damage. What will the nurse tell this patient? a. "You must stop drinking before adequate treatment can begin." b. "You must take isoniazid with close monitoring of hepatic function." c. "You must take rifampin daily for 4 months." d. "You will begin a regimen of isoniazid and rifampin."

b. "You must take isoniazid with close monitoring of hepatic function."

Which patient should begin treatment for tuberculosis? a. A patient with HIV and a tuberculin skin test result of a 4-mm region of induration b. A recent immigrant from a country with a high prevalence of TB with a 10-mm region of induration c. A patient with no known risk factors who has a job-related tuberculin skin test result of a 12-mm area of induration d. An intravenous drug abuser with a tuberculin skin test result of a 5-mm region of induration

b. A recent immigrant from a country with a high prevalence of TB with a 10-mm region of induration

A patient is about to begin treatment for latent tuberculosis with a short course of daily rifampin. The patient asks why rifapentine [Priftin] cannot be used, because it can be given twice weekly. What will the nurse tell this patient about rifapentine? a. It is more toxic than rifampin. b. It is not approved for treatment of latent TB. c. It is not well absorbed and thus not as effective. d. It will stain contact lenses orange.

b. It is not approved for treatment of latent TB.

A patient comes to a clinic for tuberculosis medications 2 weeks after beginning treatment with a four-drug induction phase. The patient's sputum culture remains positive, and no drug resistance is noted. At this point, the nurse will expect the provider to: a. change the regimen to a two-drug continuation phase. b. continue the four-drug regimen and recheck the sputum in 2 weeks. c. obtain a chest radiograph and consider adding another drug to the regimen. d. question the patient about adherence to the drug regimen.

b. continue the four-drug regimen and recheck the sputum in 2 weeks.

A patient who has drug-sensitive tuberculosis has completed 2 months of the standard four-drug therapy and asks the nurse how long he will have to take medication. Which response by the nurse is correct? a. "As long as you remain symptomatic, you will not have to take more medication." b. "The four-drug regimen will continue for 3 more months." c. "You will have to take maintenance drugs indefinitely." d. "You will need to take only two drugs for the next 4 months."

d. "You will need to take only two drugs for the next 4 months."

A patient is beginning treatment for active tuberculosis (TB) in a region with little drug-resistant TB. Which treatment regimen will be used initially? a. Isoniazid and pyrazinamide b. Isoniazid, pyrazinamide, and ethambutol c. Rifampin, pyrazinamide, and ethambutol d. Isoniazid, rifampin, pyrazinamide, and ethambutol

d. Isoniazid, rifampin, pyrazinamide, and ethambutol

A nurse is performing a physical assessment on a patient with tuberculosis who takes rifampin [Rifadin]. What would be an expected finding? a. Crystalluria b. Myopathy c. Peripheral neuropathy d. Red-orange-tinged urine

d. Red-orange-tinged urine

A patient is about to begin treatment with isoniazid. The nurse learns that the patient also takes phenytoin [Dilantin] for seizures. The nurse will contact the provider to discuss: a. increasing the phenytoin dose. b. reducing the isoniazid dose. c. monitoring isoniazid levels. d. monitoring phenytoin levels.

d. monitoring phenytoin levels.

A patient who is taking drugs to treat HIV has tuberculosis and has been on a four-drug regimen for 3 months without improvement in symptoms. Which drug will the nurse anticipate that the provider will add to this patient's regimen? a. bedaquiline [Sirturo] b. capreomycin [Capastat Sulfate] c. ethionamide [Trecator] d. pyridoxine

d. pyridoxine

A nurse reads a tuberculin skin test on a patient and notes a 6- to 7-mm area of induration. The patient is a young adult who has recently immigrated from a country with a high prevalence of tuberculosis. The patient has no other risk factors. The nurse will expect the provider to: a. begin treating this patient with a two-drug regimen of isoniazid and rifampin. b. order a chest radiograph and a sputum culture to assess for active tuberculosis. c. order a nucleic acid amplification test of the patient's sputum. d. reassure the patient that this is not considered a positive test result.

d. reassure the patient that this is not considered a positive test result.


Related study sets

Endocrine system (Control of Blood Calcium Levels)

View Set

SIE QUESTIONS MISSED ON PRACTICE TESTs 1 And 2

View Set

Family as a Social Institution Test #2

View Set